Science topic

Condensed Matter Physics - Science topic

Condensed matter physics is a branch of physics that deals with the physical properties of condensed phases of matter.
Questions related to Condensed Matter Physics
  • asked a question related to Condensed Matter Physics
Question
4 answers
The second law of thermodynamics fears experimental verification under extreme conditions, while relativity welcomes it.
Extreme conditions often validate the correctness of physical theories, while general relativity favors black holes, neutron stars, and large-scale experimental data in the universe. All of these prove the correctness of general relativity, and quantum mechanics is no exception.
The second law of thermodynamics favors simple, non quantitative empirical phenomena. Diffusion, heat conduction, frictional heat generation, rolling dice. High pressure, critical point, low-temperature thermal properties, the second law of thermodynamics, and experiments do not comply. Scientists find many reasons: due to fluctuations, uncertainty, hydrogen bonding, and biased data fitting. Science has developed for hundreds of years, and the technical problems of measurement and data processing have long been solved.
3. Compare the second law of thermodynamics with general relativity. The second law of thermodynamics requires theoretical and experimental agreement under any extreme conditions for this theory to hold true.
Relevant answer
Answer
Note that the vaporization heat calculated by scientists using the second law of thermodynamics (Clapeyron equation) is labeled as an "experimental value", which conforms to the rules of using the second law of thermodynamics. However, subsequent experiments have shown that these calculated values do not match the experiment, indicating that the more advanced the experimental technology, the more inaccurate the prediction of the second law of thermodynamics becomes.
  • asked a question related to Condensed Matter Physics
Question
4 answers
This is a ZFC/FC curve of a chromium based Mxene sample
Relevant answer
Answer
1. From Curie-Weiss fitting in high temperature region, the information that can be extracted are following:
a. Nature of magnetic interaction in ground state, from the sign of paramagnetic Curie temperature
b. If paramagnetic Curie temperature is much greater than magnetic ordering temperature, the system is accompanied with disorder. In such disorder system, high magnetic field properties are too much interesting.
c. Effective magnetic moment in the system. From effective magnetic moment, one can get an idea about how the magnetic atoms in this compound contributes to the magnetization.
2. From low temperature data, it is concluded that the system has multiple magnetic transitions, i.e. the low temperature magnetic structure is quite complex. The system is not a simple ferromagnet or antiferromagnet system. To get exactly the magnetic structure, one can go for powder neutron diffraction.
3. The FC-ZFC bifurcation, commonly known as thermomagnetic irreversibility, can be connected to various factors including domain wall pinning, presence of quenched disorder, magnetic anisotropy, or glassyness.
  • asked a question related to Condensed Matter Physics
Question
10 answers
Scientists believe that entropy is the direction that governs thermal motion, erasing the dominant role of classical physical science (determinism, entropy reduction).
1. Linear electromagnetic oscillator (LC circuit, CO2 molecule, H2O molecule): Energy storage and conversion have frequency characteristics. This depends on classical physical science, which to some extent governs the direction of thermal motion.
2. The spatial distribution of linear electromagnetic oscillators can be artificially arranged (in the technical field), affecting the spatial and temperature distribution of thermal energy (generating temperature differences), which violates the second and zero laws of thermodynamics.
3. Scientists believe that entropy is the direction that governs thermal motion, erasing the dominant role of classical physical science (determinism, entropy reduction).
4.See image for details
5.Mechanical oscillators can also generate temperature differences and entropy reductions, please refer to the link for details
Relevant answer
Answer
Hello Bo Miao,
I don't understand the specific physical system that you presented, but remember that entropy increase refers to the system + surroundings; not just the system alone that someone may be focusing their attention on. So if you have energy input into your system, your system is by definition not isolated; and you must consider the surroundings that include the entropy change needed to produce this input energy.
  • asked a question related to Condensed Matter Physics
Question
3 answers
Water is at the triple point, and the working fluid includes water, ice, and water vapor. The system volume remains unchanged, maintaining energy exchange with the large heat source while keeping the temperature constant. The equilibrium state has the following equation:
mg+ml+ms=m0
mgg+mll+mss=V
There are two equations, three unknown variables,, and an infinite number of equilibrium solutions in a system. The three-phase distribution of matter cannot be determined, and thermodynamics requires that the equilibrium state of a system be unique, which is inconsistent with physical facts. The thermodynamic direction advocated by the second law of thermodynamics will also be lost, such as volume expansion, which cannot determine whether it is the evaporation of liquid water or the transformation of ice into water vapor.
Relevant answer
Answer
...the very words you use ('large heat source') contradict what you say. There's a third very definite equation that stands behind it.
  • asked a question related to Condensed Matter Physics
Question
17 answers
This happened in the teaching practice of the second law of thermodynamics, where they taught students that Carnot efficiency is independent of the working fluid, and that there are physical concepts that can be separated from physical entities. The following is a specific analysis:
Thermal engine system: pistons, containers (geometric boundaries), heat sources (energy boundaries), etc. are all centered around the working fluid, and calculating thermal power conversion is studying the working fluid. The Carnot efficiency is independent of the working fluid, and is detached from physical entities, becoming a concept suspended in the air.
2. This one is wrong, the entire second law of thermodynamics is incorrect.
3. Entropy is the logical successor of Carnot's law and cannot be used to justify it here.
4. Carnot efficiency is reversible thermodynamics and irreversibility cannot be used to justify it here.
Relevant answer
Answer
Bo Miao Sandeep Jaiswal Jixin Chen & ResearchGate Community
  • Dear Bo and Sandeep Jaiswal,
  • Reevaluating the Second Law of Thermodynamics The notion that entropy can remain constant in open systems is a compelling area of investigation. While the Second Law posits that entropy in isolated systems tends to increase, open systems—which exchange energy and matter with their surroundings—can indeed exhibit dynamic behaviors that deviate from this principle. However, the constancy of entropy in such systems is contingent on specific conditions, such as achieving a dynamic equilibrium between incoming and outgoing flows. This equilibrium is rare and context-dependent, making it more of an exception than a general rule. Therefore, asserting that the Second Law "does not universally apply" necessitates a precise delineation of the conditions under which this claim holds.
  • Heat Engine Efficiency and Carnot's Principle The challenge to Carnot efficiency based on the influence of the working substance is an intriguing perspective. Carnot's formulation assumes an idealized, lossless system and asserts that the maximum efficiency of a heat engine depends solely on the temperatures of the reservoirs, not the working substance. While real-world systems often deviate from these ideal conditions due to factors like viscosity and thermal conductivity, concluding that the working substance fundamentally alters Carnot efficiency would require rigorous experimental and theoretical evidence. Until such evidence is provided, Carnot's efficiency remains a cornerstone of thermodynamic theory, serving as a theoretical upper limit rather than a practical guarantee.
  • The Arrow of Time and Restoring Time Symmetry The suggestion that time symmetry can be restored in open systems with constant entropy is a fascinating proposition. The arrow of time is deeply rooted in the statistical nature of entropy increase, as articulated by the Second Law. If a system can maintain constant entropy over time, it raises profound questions about the relationship between time's unidirectional flow and entropy. However, it is essential to distinguish between the philosophical and physical interpretations of time's arrow. Any proposed restoration of time symmetry must align with empirical evidence and the foundational principles governing irreversible processes in macroscopic systems.
  • Conclusion Your research presents a thought-provoking challenge to conventional thermodynamic principles, particularly regarding open systems. While these ideas hold the potential to expand our understanding, the conclusions require robust mathematical modeling, experimental validation, and alignment with established physical laws. I look forward to engaging further and exploring the specific methodologies and results from your work.
  • asked a question related to Condensed Matter Physics
Question
4 answers
The working fluid is the core of the heat engine, and the Carnot efficiency is independent of the working fluid (core). Do you believe in the second law of thermodynamics?
Relevant answer
Answer
It's a bit illogical, I won't argue anymore. Carnot efficiency is independent of the working fluid -—— the second law of thermodynamics violates common sense and is lower than the average level of human intelligence.
  • asked a question related to Condensed Matter Physics
Question
4 answers
Conclusion of the 2nd law of thermodynamics is that the Carnot efficiency of carbon dioxide, water vapor, liquid water, solid water... is 1-T2/T1. Do you believe it?
Relevant answer
Answer
You have been given the original formulations by the original authors of the second law and a logical deduction above. If your textbooks convert this into an odd quantitative statement, their authors or translators did a bad job.
The actual second law is, as shown in my previous post, a pretty basic statement, so what you are fighting against has actually never been the second law. i get that after all the time you wasted on this that is hard to grasp.
The quantified threshold (not prediction) of the second law with respect to the Carnot cycle is:
"Es gibt keine Wärmekraftmaschine, die bei gegebenen mittleren Temperaturen der Wärmezufuhr und Wärmeabfuhr einen höheren Wirkungsgrad hat als den aus diesen Temperaturen gebildeten Carnot-Wirkungsgrad."
There is no heat engine, which at given mean temperatures of of heat input and output provides a higher efficiency than the Carnot efficiency calculated from these temperatures.
  • asked a question related to Condensed Matter Physics
Question
5 answers
  1. The calculation results of the first and second laws of thermodynamics are different.
  2. The calculation method of the second law of thermodynamics involves data piecing together, resulting in a compromise between theory and experiment. It is believed to cause differences between the two, and this method is widely adopted.
  3. If the second law of thermodynamics does not use the piecing together method, its deviation from the experiment will be exposed. This patchwork method is meant to conceal this deviation. It is a shameful behavior.
Relevant answer
Answer
Science speaks in mathematics, and to prove the correctness of the second law of thermodynamics, quantitative predictions must be made and consistent with experiments.
  • asked a question related to Condensed Matter Physics
Question
30 answers
According to the logic of the second law of thermodynamics, it can be inferred that a person's grades are not related to their intelligence. Do you believe it? The specific derivation is as follows
1. Carnot efficiency (thermal engine function) is independent of the working fluid (thermal engine soul).
2. The function of a heat engine is not related to its soul.
3. Analogous to humans, it can be concluded that a person's academic performance is not related to their intelligence.
4,Do you still believe that Carnot efficiency (thermal engine function) has nothing to do with the working fluid (thermal engine soul).?Is this a low IQ perception?
Relevant answer
Answer
  1. The calculation results of the first and second laws of thermodynamics are different.
  2. The calculation method of the second law of thermodynamics involves data piecing together, resulting in a compromise between theory and experiment. It is believed to cause differences between the two, and this method is widely adopted.
  3. If the second law of thermodynamics does not use the piecing together method, its deviation from the experiment will be exposed. This patchwork method is meant to conceal this deviation. It is a shameful behavior.
  • asked a question related to Condensed Matter Physics
Question
1 answer
  • The second law of thermodynamics creates mathematical paradoxes by substituting formulas in mathematical calculations.
  • The second law of thermodynamics can only study the temperature of the heat source and cannot study the working fluid. Please refer to the attached diagram for details
Relevant answer
Answer
Only some misunderstanding of the 2nd law of thermodynamics can cause any paradox.
It simply states that any change of state of any stable system requires an expenditure of some amount of energy to account for the change.
  • asked a question related to Condensed Matter Physics
Question
3 answers
XTLS Code is widely used to model the photoelectron spectra through full-multiplet configuration-interaction calculations. However, I couldn't find any web page related to the code. Is this code package still available?
Relevant answer
Answer
Late answer, but Crispy is another good alternative
  • asked a question related to Condensed Matter Physics
Question
1 answer
“actual decomposition voltage < reversible decomposition voltage” indicates an error in the 2nd law of thermodynamics.Please refer to the attached diagram for details.
I hope everyone respects the experiment
Relevant answer
Answer
The reversible voltage is measured at zero current. The actual decomposition voltage depends on the current density and contains contributions from electric resistance (Ohm's Law) and reaction kinetics
(so-called overvoltage) which are not covered by the 2nd Law. The difference of experimental decomposition voltages and reversible voltages is not an argument against the 2nd Law.
  • asked a question related to Condensed Matter Physics
Question
1 answer
C.N.Yang vs Carnot: mathematical symmetry extension vs super empirical fantasy
By comparison, help everyone break free from the empirical quagmire of the second law of thermodynamics. See the picture for details.
Relevant answer
Answer
Your question can have a sens for a system at T=zero Kelvin. A molecule at this temperature don't oscillate but can spin in three sens or much more for a Universe with D-spatial dimensions.
  • asked a question related to Condensed Matter Physics
Question
10 answers
The first law of thermodynamics replaces the second law of thermodynamics.Please refer to the attached diagram for details
Relevant answer
Answer
Thank you. Your answers clearly demonstrate your level of discussion.
  • asked a question related to Condensed Matter Physics
Question
5 answers
The second law of thermodynamics switches formulas surreptitiously in mathematical calculations.
For details, see the attached figure.
Relevant answer
Answer
Q=f (T) is the mathematical expression of "Carnot efficiency is only related to the temperature of the heat source". DQ=df (T) The second law of thermodynamics in mathematics (S) is derived from this and the definition of dQ cannot be changed arbitrarily.
In fact, the second law of thermodynamics changes the mathematical representation of dQ.
  • asked a question related to Condensed Matter Physics
Question
11 answers
Abstract: There is heat exchange between two real gases at the same temperature. According to the first law of thermodynamics, the temperature changes of V1 and V2 in adiabatic cycles are not equal to 0. When the temperature change of the cycle is less than 0, thermal work conversion is achieved. Set a single heat source and restore the initial temperature T0 of the system. This is the second type of perpetual motion machine.
Please refer to the attached diagram for details
  • asked a question related to Condensed Matter Physics
Question
5 answers
I refered two books, but they gave me the different answers. One said the lower the pressure, the thicker the boundary layer. The other said the oppsite result. 
Relevant answer
Answer
My experiences from large regenerative heat exchangers are that thick boundary layers (i,e. turburbulent flow) increase the pressure drop.
But Fanning friction factor goes down.
  • asked a question related to Condensed Matter Physics
Question
9 answers
The foolish logic of the second law of thermodynamics (Kelvin's argument): I am against the second type of perpetual motion machine, so I am right. Please refer to the attached diagram for details.
Relevant answer
Answer
The last part is the first and second laws of thermodynamics work together, and you can’t swap one for the other. The first law focuses on energy conservation—it tells us how much energy is moving around, whether it's being used or stored. The second law, however, tells us about the quality of that energy—whether it can still be used to do work, the natural direction of processes, and why energy is always lost in the form of heat or friction.
For example, in a heat engine, the first law can tell us how much energy is being used or lost, but only the second law can explain why the engine can never be 100% efficient and why some energy is always wasted. Ignoring the second law would mean losing these important insights into how real-world processes work.
  • asked a question related to Condensed Matter Physics
Question
9 answers
The second law of thermodynamics: η=1-T1/T2 is only a physical 0-order approximation.(Interaction is castrated) as shown in the picture
Relevant answer
Answer
Your "comprehensive" article is based on the same false assumption as this one. Read Planck's original book, it's free at Project Gutenberg and, as you said yourself, Germany in general and Planck in particular are the original sources, so he knows what assumptions he made. If you base your life on bad resources and don't want to see that, that's not the fault of my countrymen...
  • asked a question related to Condensed Matter Physics
Question
2 answers
The 2nd law of thermodynamics is a conjecture about the efficiency of a heat engine that deviates from reality and has become witchcraft.
Comparing the first and second laws of thermodynamics when studying heat engines, you will find that the second law of thermodynamics is purely speculative.See the picture for details.
Relevant answer
Answer
The question highlights the difference between how Carnot efficiency is studied in the second law of thermodynamics and how engineers approach practical thermal systems.
Writing:
The second law of thermodynamics provides a theoretical framework for understanding how thermal systems work and sets the upper limit for efficiency in converting heat into work. This theoretical limit is called "Carnot efficiency," which represents the maximum possible efficiency a heat engine can achieve when operating between two thermal reservoirs (a hot and a cold reservoir). Carnot efficiency does not require detailed knowledge of the relationships between pressure, volume, and temperature. It simply depends on the absolute temperatures of the reservoirs and is calculated as:
\[
\eta_{Carnot} = 1 - \frac{T_2}{T_1}
\]
where T₁ is the temperature of the hot reservoir and T₂ is the temperature of the cold reservoir. This equation represents the theoretical upper bound on efficiency, assuming the engine operates in an ideal, reversible manner without any energy loss.
However, this theoretical picture often differs from the practical reality that engineers face. In real-world applications, engineers need to go beyond just knowing the theoretical limits of efficiency; they must also understand how the system behaves in terms of detailed thermodynamic properties. They use equations like \( P = P(V, T) \) (pressure as a function of volume and temperature) and \( E = E(V, T) \) (energy as a function of volume and temperature) to design systems that approach these limits while accounting for real-world inefficiencies and material behavior.
In short, while the second law and Carnot efficiency provide the theoretical limits, engineers must deal with the practical aspects of system design, using detailed relationships between pressure, energy, volume, and temperature to optimize the system within those theoretical constraints.
  • asked a question related to Condensed Matter Physics
Question
3 answers
Using python codes, the output image lags so much that I am unable to tune the code.
Relevant answer
Answer
If you have done DFT calculations, you can use the wannier90 code to calculate the wannier function, and use the xcrysden software to plot a 3-dimensional image of the s,p,d orbitals.
  • asked a question related to Condensed Matter Physics
Question
3 answers
  • The second law of thermodynamics states that the thermodynamic entropy of an isolated system (dQ=0, ds=dQ/T=0) is constant. Statistics S=k * In (W), is it an increase?
  • The second law of thermodynamics : statistical entropy(S=k*In(W)) of an isolated system increases, while thermodynamic entropy(ds=dQ/T) remains constant.
  • Thermodynamic entropy (ds=dQ/T) is not equivalent to statistical entropy {S=k * In (W)}
Relevant answer
Answer
"Be more complete and logical." – Well, your statements are incomplete.
"Don't solve problems just because of a few trendy terms." – Well, please remember Clausis' famous statement about entropy (cf-, e.g., https://en.wikipedia.org/wiki/Clausius_theorem); it is an inequality, so one has to consider two different cases.
YOU started this discussion; I'm not here to do your homework.
  • asked a question related to Condensed Matter Physics
Question
3 answers
E - has spatiotemporal continuity, S - is statistical and does not have spatiotemporal continuity. Is it correct to write E = F + S*T together?
The second law of thermodynamics is statistical.S - is statistical and does not have spatiotemporal continuity.
It's hard to imagine a concept that exists outside of time and space.
Relevant answer
Answer
E is statistical to the same amount to which S is statistical, but since the trends in E can quite often be understood without explicitely invoking statistics, it is just skipped quite often in trend explanations.
E and S are both extensive quantities, so there is no spatiatemporal issue.
  • asked a question related to Condensed Matter Physics
Question
3 answers
  • The term 'f=ma' in the figure refers to classical physics, electromagnetics, relativity, and quantum mechanics
  • Thermodynamics and statistical physics are the results of "f = ma",
  • "f = ma" supports the second type of perpetual motion machine.
  • The opposition of thermodynamics to the second type of perpetual motion machine is not in line with logic.
  • For details, please refer to the picture.
Relevant answer
Answer
You are way too full of all kinds of misconceptions.
Start with a full intoductory physics
Series as the Berkly or Feyman.
  • asked a question related to Condensed Matter Physics
Question
2 answers
I am attempting to reproduce phonon surface state calculations using the Wannier tools package. In the first step, I aim to obtain the phonon tight binding Hamiltonian from force constants or force sets by running the phonon_hr.py program. However, I encounter an error message stating: "cannot import name 'create_FORCE_SETS' from 'phonopy.interface' (.../python3.8/site-packages/phonopy/interface/init.py)".
Despite having successfully installed Phonopy, this error persists. Upon commenting out the line "from phonopy.interface import create_FORCE_SETS, read_crystal_structure" from the code, a new error arises: options, args = parser.parse_args() AttributeError: 'tuple' object has no attribute 'parse_args'.
I would greatly appreciate any insights or guidance on resolving these issues and successfully obtaining the phonon tight-binding Hamiltonian.
Relevant answer
Answer
Thank you sir for your response.
  • asked a question related to Condensed Matter Physics
Question
3 answers
The second law of thermodynamics, including Carnot's law, is self-contradictory. For details, please refer to the picture. France is inviting scientists from all over the world to commemorate this self-contradictory theory. Isn't it funny and ironic?
One more Carnot's celebration: https://carnot-legacy.sciencesconf.org/
The colloquim, focusing on modern thermodynamics, will take place on the week following Carnot Lille 2024, which follows a more historical focus on Sadi Carnot and his publication.
Relevant answer
Answer
As multiple people have already informed you, your schematic has the error that the material independent efficiency is not a fixed value, but constitutes a maximum that you cannot overcome independent of the material, so the contradiction is non-existent.
But thanks for the new entertainment.
  • asked a question related to Condensed Matter Physics
Question
4 answers
The blackbody cavity contains CO2, and the blackbody radiation contains the characteristic spectrum of CO2, which does not satisfy the Planck formula.
  • There is CO2 inside the blackbody cavity, and radiation enters from point A with an absorption rate of 1,meets the definition of blackbody.
  • The energy density of the characteristic spectrum of CO2 inside the cavity will increase, and the outward radiation density will no longer be Smooth Planck's formula: a characteristic spectrum containing CO2.
  • The emissivity is no longer equal to 1, and varies with different filling gases.
  • Blackbodies with different emissivities emit heat from each other, resulting in temperature differences and the failure of the second law of thermodynamics.
  • See image for details
Relevant answer
Answer
The blackbody cavity filled with CO₂ gas. This situation introduces additional complexities to the standard blackbody radiation model, which is typically based on an idealized cavity with no interactions with gases or other materials inside it. Here are some points to consider:
Blackbody Radiation and Planck's Law
  • Ideal Blackbody: An ideal blackbody absorbs all incident radiation and re-emits it according to Planck's law, which depends only on the temperature of the blackbody and is independent of the material.
  • Planck's Formula: For an ideal blackbody at temperature TTT, the spectral radiance B(ν,T)B(\nu, T)B(ν,T) is given by: B(ν,T)=8πν2c3hνehν/kT−1B(\nu, T) = \frac{8 \pi \nu^2}{c^3} \frac{h \nu}{e^{h \nu / k T} - 1}B(ν,T)=c38πν2​ehν/kT−1hν​where ν\nuν is the frequency, ccc is the speed of light, hhh is Planck's constant, and kkk is Boltzmann's constant.
Influence of CO₂ Gas in the Cavity
  • Absorption and Emission Lines: CO₂ molecules have specific absorption and emission lines in the infrared region due to their vibrational and rotational transitions.
  • Non-Ideal Spectrum: The presence of CO₂ gas means that the radiation spectrum will show characteristic absorption and emission lines superimposed on the blackbody spectrum. These spectral lines correspond to the specific energy level transitions of the CO₂ molecules and deviate from the continuous spectrum predicted by Planck's law.
Modified Spectrum
  • Characteristic Spectrum of CO₂: The spectrum will contain peaks (emission lines) and dips (absorption lines) at wavelengths corresponding to the vibrational and rotational transitions of CO₂ molecules. This modified spectrum does not match the continuous blackbody spectrum given by Planck's law.
  • Thermal Equilibrium: If the CO₂ gas and the cavity walls are in thermal equilibrium, the gas molecules will emit and absorb radiation in a way that can still be described by Planck's law at a macroscopic level, but with the detailed structure of the CO₂ spectrum visible.
Understanding the Deviation
  • Spectral Lines Impact: The deviations from the Planck spectrum are due to the discrete energy levels of CO₂ molecules. These deviations manifest as specific spectral lines, which are not accounted for in the ideal blackbody radiation model.
  • Line Broadening: In real situations, these lines may also be broadened due to various effects such as Doppler broadening and pressure broadening, which can further modify the observed spectrum.
  • asked a question related to Condensed Matter Physics
Question
3 answers
Radiation perpetual motion machine: uses radiation pressure to do work and consume heat energy. ---Radiation is remote energy transfer. See image for details
  • Two identical small buckets are arranged symmetrically, with openings facing each other. The radiation rate at the bottom of the bucket is ε=1, and the rest is ε=0.
  • The two bottoms radiate energy and absorb radiation:q=εσT^4*S.
  • The force acting on the small bucket is: F=2εσT^4*S/C.
  • The speed of the small bucket increases, the kinetic energy increases, and the temperature decreases.
  • There is no limitation of the second law of thermodynamics for thermal conversion, and the second law of thermodynamics is invalid.
Relevant answer
Answer
When compression occur molecules get slide over each other due to which friction occur which results in rising of phonons or heat is produced. The temperature of system increases.
  • asked a question related to Condensed Matter Physics
Question
3 answers
  • As shown in the figure: Use a transparent solid to separate 3mol/L and 1mol/L of CO2, allowing the gases to radiate each other.
  • Radiation energy is transferred from container A (3mol/L) to container B (1mol/L).
  • Temperature: Tb>Ta
  • Scientists love to use thermal diffusion and heat transfer to explain the second law of thermodynamics, which is the result of short-range interactions. Radiation is a long-range interaction that reaches the macroscopic scale, making it easy for people to control the direction of energy transfer.
  • Solid or liquid (doping) can also be used to artificially create asymmetric radiation and control the direction of energy transfer.
Relevant answer
Answer
The concept of perpetual motion machines has fascinated scientists and inventors for centuries. However, it’s essential to understand that true perpetual motion machines are impossible due to the laws of thermodynamics. Let’s break down the scenario you’ve described and address the researcher’s question.
A perpetual motion machine is a hypothetical device that can perform work indefinitely without an external energy source. Such a machine would violate the laws of thermodynamics. The laws of thermodynamics apply universally, regardless of the system’s size or complexity. There are three types of perpetual motion machines:
Machines of the First Kind: These machines produce work without any energy input. They violate the first law of thermodynamics. Machines of the Second Kind: These machines spontaneously convert thermal energy into mechanical work without any input. While they don’t violate the conservation of energy, they do violate the second law of thermodynamics. Machines of the Third Kind: These machines continue to be in motion forever due to inertia, but they cannot exist in practice due to unavoidable dissipation (e.g., friction).
In your scenario, you have two containers (A and B) separated by a transparent solid. Container A contains CO2 at a concentration of 3 mol/L, while container B contains CO2 at 1 mol/L. Radiation energy is transferred from container A to container B. The temperature in container B (Tb) is higher than in container A (Ta).
You are interested in whether this setup could lead to a perpetual motion machine based on radiation. Unfortunately, this scenario does not solve the problem of perpetual motion. Here’s why:While radiation is a long-range interaction, it still obeys the laws of thermodynamics.
The second law of thermodynamics states that the entropy (disorder) of an isolated system tends to increase over time. In other words, energy spontaneously flows from hotter regions to cooler regions. In your setup, container A (with higher concentration) will naturally radiate energy to container B (with lower concentration), resulting in cooling of container A. This process cannot continue indefinitely without an external energy source.
The temperature difference (Tb > Ta) does not change this fundamental limitation.
Even if you were to achieve perfect control over radiation, the energy stored in the CO2 concentrations would eventually be exhausted. Perpetual motion machines remain theoretical and cannot be commercialized because they violate the laws of thermodynamics.
In summary, while the concept of using radiation for energy transfer is intriguing, it does not provide a solution to perpetual motion. Researchers should focus on practical and sustainable energy solutions that adhere to the fundamental principles of physics
  • asked a question related to Condensed Matter Physics
Question
3 answers
Scientists have abandoned experimental proof of η=1-T1/T2 and instead used experimental data to piece together η=1-T1/T2. Shameful! Please refer to the attached diagram for details:
1) Method A in the figure is a method for verifying Carnot efficiency, which scientists rarely use because the experiment deviates significantly from theoretical predictions.
2) Scientists extensively use method B in the figure, which does not involve theoretical predictions, but instead uses experiments to gather theory. This is shameful, it's data fraud. The enthalpy entropy charts we use are all pieced together using method B.
3) Scientists explain why method A is not necessary. It's because the experiment is not good, which is deceiving. Science has developed for hundreds of years, and even more rare experiments can be conducted.
4) The core is that scientists are unwilling to admit that the second law of thermodynamics is inconsistent with experiments, but this violates scientific discipline and morality.
Relevant answer
Answer
Everyone can compare and read to see who is bragging.
  • asked a question related to Condensed Matter Physics
Question
3 answers
I would like to explain my question with the following illustrative situation. In general, when we apply the high-pressure to the solid materials at room temperature conditions, the solid materials undergoes phase transformation.
For instance, InTe compound adopts the tetragonal structure (space group: I4/mcm) at room temperature and room pressure conditions. When the isothermal hydrostatic pressure (i.e., keeping temperature constant, and applying the pressure), this solid material, transform into face centered cubic phase (space group: Fm-3m) at ∼6.0GPa. Upon further pressure increase, this material again transform into primitive cubic phase (space group: Pm-3m) at P ∼ 15 GPa.
When we release the pressure, this material come back to original initial structure (tetragonal: I4/mcm) at room temperature and room pressure. The pressure induced phase transition is completely reversible.
In this case, can we say that, I synthesized the InTe compound with Fm-3m and Pm-3m structures?
Is it scientifically correct to use the synthesis word in this context?
Please let me know.
Your valuable explanation, suggestion, and guidance will be very useful to our research works. Thank you very much in advance.
Relevant answer
Answer
Sure senior,
Well, you are not able to lock the phase hence the reversibility is occurring, or the material cannot withstand such interactions (in different phase symmetry) under ambient conditions.
So, In this case, you might have probed the mechanical stability/effect on the material but you have not 'synthesized' a stable product.
  • asked a question related to Condensed Matter Physics
Question
2 answers
The picture is a screenshot of the literature: 1. It illustrates the Crabelon equation derived from the second law of thermodynamics: the calculated heat of vaporization does not match the experiment.
The more precise the experiment, the more obvious the inconsistency between the second law of thermodynamics and the experiment.
3. Why would this happen? The second law of thermodynamics violates symmetry and conservation (which is the mainstream of natural science).
4,Some scientists are packaging the experimental deviations of these theories, and the data in the enthalpy entropy chart that everyone sees is completely consistent with the second law of thermodynamics, which is a deceptive illusion.
Relevant answer
Answer
The second law of thermodynamics is a fundamental principle in physics that describes the direction of natural processes and sets limitations on the efficiency of heat engines. It can be stated in various forms, but one common statement is that in any spontaneous process, the total entropy of a closed system plus its surroundings will increase.
While the second law has been extraordinarily successful in explaining and predicting the behavior of systems, it's worth noting that there can be challenges in applying it universally to every situation, particularly at the microscopic level or in systems that are far from equilibrium.
One reason for potential inconsistencies with experiments could be the difficulty in precisely defining and measuring quantities like entropy, especially in systems with many particles or complex interactions. Entropy is a measure of the disorder or randomness of a system, and its calculation often relies on statistical mechanics, which involves making statistical assumptions about the behavior of particles.
Additionally, at the microscopic level, there are instances where the behavior of individual particles may seem to violate the second law, leading to discussions about statistical fluctuations and the arrow of time. However, these apparent violations typically occur on very short timescales or involve very small systems and do not contradict the overall validity of the second law for macroscopic systems.
As for symmetry and conservation, the second law doesn't strictly require these properties. Symmetry and conservation principles are important in physics, but the second law is more about the directionality of processes rather than symmetries or conservations.
Overall, while the second law of thermodynamics has been extremely successful in describing the behavior of macroscopic systems, there are still ongoing debates and research efforts to understand its implications at smaller scales or in more complex systems. These discussions often lead to deeper insights into the nature of thermodynamics and the behavior of physical systems.
  • asked a question related to Condensed Matter Physics
Question
12 answers
1,The image comes from the scientific classic "the propeties of Gases and Liquids"
The picture is a screenshot of the literature: 1. It illustrates the Crabelon equation derived from the second law of thermodynamics: the calculated heat of vaporization does not match the experiment.
2,The more precise the experiment, the more obvious the inconsistency between the second law of thermodynamics and the experiment.
3,Even in the face of such facts, scientists still confidently persist.
4,The second law of thermodynamics does not conform to experiments, and scientists use it to make money is a scam.
Relevant answer
Answer
Solution to this one:
  • expansion into vacuum has a dW
  • 1st LoT: dW=-dQ
  • So dQ is not 0
So once more you have shown that if you violate the first law, you also violate the second law.
  • asked a question related to Condensed Matter Physics
Question
4 answers
The actual decomposition voltage is less than the reversible decomposition voltage, which violates the second law of thermodynamics.
1)Experiments show that there exists an actual decomposition voltage which is less than the reversible decomposition voltage. ==》
2)The change of Gibbs free energy is related to the path.==》
3)The cyclic integral of entropy is not zero.==》
4)The second law of thermodynamics becomes invalid.
5) Scientists can only pretend to be deaf and dumb to such a fact.
Please see the picture for details.
Relevant answer
Answer
The second law of thermodynamics contradicts itself. Scientists are also addicted to the surface of experience.
Galileo's introduction of Aristotle's theory of falling is contradictory, and Aristotle's theory of falling is ineffective.
  • asked a question related to Condensed Matter Physics
Question
4 answers
Abstract: There is gas in the container: CO, O2, CO2, and solid C, among which there are four equations: three chemical equilibrium equations and one element conservation equation, but there are only three variables: the densities of three kinds of gases. In this way, the number of equations is greater than the number of variables, there is no solution, an isolated system has no equilibrium state, and the second law of thermodynamics fails.
See image for details
Scientists can determine the truth of the second law of thermodynamics as long as they can count. Come on!
Relevant answer
Answer
Dear Bo Miao ,
I understand your concern, as at first glance, it seems like there are more equations than variables, which could imply that there is no solution. However, upon a more rigorous mathematical analysis, I believe the second law still holds in this case.
In your system, you have CO, O2, CO2 gases, and solid carbon (C) with three chemical equilibrium reactions: 2CO + O2 ⇌ CO2, C + O2 ⇌ CO2, and C + CO2 ⇌ 2CO. The corresponding equilibrium equations are ρ_co^2 / (ρ_co2 * ρ_o2) = K1, ρ_co2 / ρ_o2 = K2, and ρ_co^2 / ρ_co2 = K3. Additionally, you have an element conservation equation, 0.5ρ_co + ρ_co2 + ρ_o2 = A, where A is the average concentration of O2.
The equilibrium constants K1, K2, and K3 are not independent variables. They are related to the Gibbs free energy change (ΔG) of each reaction at a given temperature T by the equation ΔG = -RT ln(K), where R is the gas constant. Therefore, only two of the three K values are independent.
We can rewrite the equilibrium equations in terms of ΔG: ΔG1 = -RT ln(K1) = -RT ln(ρ_co^2 / (ρ_co2 * ρ_o2)), ΔG2 = -RT ln(K2) = -RT ln(ρ_co2 / ρ_o2), and ΔG3 = -RT ln(K3) = -RT ln(ρ_co^2 / ρ_co2). The sum of the ΔG values for the three reactions must be zero, as the overall process is at equilibrium: ΔG1 + ΔG2 + ΔG3 = 0. Substituting the expressions for ΔG1, ΔG2, and ΔG3, we get -RT ln(ρ_co^2 / (ρ_co2 * ρ_o2)) - RT ln(ρ_co2 / ρ_o2) - RT ln(ρ_co^2 / ρ_co2) = 0.
This equation, along with the element conservation equation, forms a system of two equations with three variables (ρ_co, ρ_co2, ρ_o2). While this system is underdetermined, it does not necessarily mean there is no solution or that the second law of thermodynamics is violated. The second law states that the entropy of an isolated system at equilibrium is maximum, and the change in Gibbs free energy is zero. The equilibrium state of this system, if it exists, would satisfy these conditions.
The apparent contradiction between the number of equations and variables in this chemical system does not inherently violate the second law of thermodynamics. The equilibrium constants are related through the Gibbs free energy, reducing the number of independent equations. The system may have an equilibrium state consistent with the second law, even if it is not uniquely determined by the given equations alone. Additional constraints, such as the positivity of concentrations or the minimization of Gibbs free energy, could potentially lead to a unique solution.
The second law of thermodynamics is a fundamental principle that has been extensively verified experimentally. I think it is important to critically examine such principles, but the presented argument does not provide sufficient evidence to overturn this well-established law.
Best,
Alessandro Rizzo
  • asked a question related to Condensed Matter Physics
Question
5 answers
I am a PhD student working in Exp. Condensed Matter Physics. I am working on some Hall Resistivity data. I have few question regarding the Anomalous Hall Effect.
1). How I will know that AHE is present in my Hall Data ?
2). If it is present, How I can extract it ?
I would be happy to if someone explain it for me.
Relevant answer
Answer
Keep in mind that scientists often call the Hall effect anomalous, when in fact it is not. The fact is that the simplest one-band model does not always explain the experiment. But if you apply a two-band model, the behaviour of this effect fits within its framework. For example, the change in the sign of the Hall effect in superconductors is treated as an anomaly, but I have shown that there is no anomaly there. I am sending you this publication. For more details, see pages 7-9 in [Yu. Uhryn, O. Kuzyk, Minority Current Carriers are Responsible for the Superconducting State, Romanian Journal of Physics 68, 606 (2023)]
  • asked a question related to Condensed Matter Physics
Question
2 answers
The Maxwell demon utilizes temperature fluctuations to achieve a perpetual motion machine, independent of information theory.
Please refer to the attached diagram and the following text for details.
1,The Maxwell demon measures the instantaneous temperature fluctuations of a and b on both sides of the switch.
2,When Ta>Tb, the switch is turned on. Heat is transferred from a to b
When Ta<Tb, the switch is turned off. a&b Insulation.
3,Finally, the temperature difference in the container: TA<TB
4,The Maxwell demon is unrelated to information theory and satisfies Newton's laws and energy conservation.
5,Fluctuations themselves violate the second law of thermodynamics. Maxwell's demon utilizes the defect of the second law of thermodynamics.
Relevant answer
Answer
these preprint papers claimed reversing entropy by mixing Raoult's law with osmosis principle and extended Gibbs Donnan Equilibrium .
What do you think about this novel approach?
Title of the papers:
Experimental Demonstration of Energy Harvesting by Maxwell's Demon Device
DOI: 10.20944/preprints202403.1698.v1
....
An Autonomous Mechanical Maxwell's Demon
DOI: 10.14293/S2199-1006.1.SOR-.PP5S6NK.v1
  • asked a question related to Condensed Matter Physics
Question
1 answer
Gas radiation intensity is a function of space: I=I (r). This is recognized in the textbook of heat transfer. Pushing forward two more steps will result in a temperature difference (this is the second type of perpetual motion machine). Please refer to the picture for details.
The second type of perpetual motion machine for gas radiation is the simplest, easy to implement, and commercialized. The wealth generated by the second type of perpetual motion machine can satisfy everyone's desires.
The current war of Russian aggression against Ukraine is trending towards a third world war. If scientists accept the second type of perpetual motion machine, they will find that the things being fought for in the war are no longer scarce, and the war will come to an end. I hope scientists can play their role.
Relevant answer
Answer
我觉得外星人来锣
  • asked a question related to Condensed Matter Physics
Question
10 answers
Gas diffuses into vacuum, dQ=0, thermodynamic entropy dS=dQ/T=0. The second law of thermodynamics cannot be calculated.Please refer to the attached diagram for details。
"Ds=dQ/T" is defined as a reversible process that can be used, but an irreversible process that cannot be used. This violates the universality and consistency of natural science.
Relevant answer
Answer
The formula change in S = dQ/T refers to a particular case, that of simple thermal heating. The expansion of an ideal gas into a vacuum involves no change in energy, so this formula does not apply to that case.
  • asked a question related to Condensed Matter Physics
Question
3 answers
Heat transfer (gas radiation) does not support the second law of thermodynamics.
Please refer to the following text and pictures for details
Gas radiation and absorption occur throughout space, and gases at different locations absorb energy differently from remote radiation. The different amount of radiation absorbed by gases at different positions can lead to temperature differences. The second law of thermodynamics is invalid.
Do scientists have to wait until nuclear war breaks out to believe in the existence of perpetual motion machines?
Relevant answer
Answer
i assume this is genuine question you have raised;
leading with that,
The statement that gas radiation and absorption could invalidate the Second Law of Thermodynamics is a misunderstanding of how thermodynamics and heat transfer operate. The Second Law of Thermodynamics, in its simplest form, states that in any closed system, the entropy (a measure of disorder or randomness) tends to increase over time unless energy is put into the system to maintain or decrease entropy. This law is one of the fundamental principles underlying much of physics and engineering and is observed to hold true in countless experimentation.
Regarding the specific points raised:
1. Gas Radiation and Absorption
it true that gases at different locations can absorb radiation differently and this can lead to temperature differences across a system. However, this process does not violate the Second Law of Thermodynamics. The law does not imply that temperature differences cannot exist or that they cannot change; it primarily concerns the overall entropy of a closed system. In the context of gas radiation, the energy transfer through radiation leads to changes in temperature and can drive processes that increase the system's overall entropy.
2. Temperature Differences and Entropy
The creation of temperature differences through radiation absorption and emission is a part of how heat transfer operates in the universe. These processes, including conduction, convection, and radiation, are mechanisms for energy distribution and do not inherently contradict the Second Law. The entropy increase or decrease in a particular part of a system does not imply a violation of the law as long as the total entropy of the closed system, when considering all interactions, does not decrease.
3. Perpetual Motion Machines
The idea of a perpetual motion machine—a machine that can operate indefinitely without an energy source—is a concept that violates the First and/or Second Law of Thermodynamics. Despite extensive theoretical and experimental exploration, no such machine has been created or observed to exist. The Second Law, among other principles, indicates why perpetual motion machines are not feasible.
I did struggle to understand the non sequiator in your comments, as per the suggestion that belief in such devices is contingent on catastrophic events like nuclear war…it is not grounded in scientific reasoning.
Scientists rely on empirical evidence and theoretical consistency to validate or refute theories. The Second Law of Thermodynamics is supported by a vast body of evidence and theoretical understanding, making it one of the cornerstones of physical science. While scientific understanding evolves with new discoveries, any new theory or observation that appears to contradict well-established laws like the Second Law of Thermodynamics requires rigorous scrutiny, experimental validation, and theoretical explanation within the broader framework of physics.
best
H
  • asked a question related to Condensed Matter Physics
Question
3 answers
Gas radiation has no thermal equilibrium, and the second law of thermodynamics is invalid. The following pictures are all from the content of heat transfer and university physics, combined together, it is found that the second law of thermodynamics is invalid.
Please refer to the picture for details.
Relevant answer
Answer
Three formulas can explain that gas radiation cannot reach thermal equilibrium, and the second law of thermodynamics is incorrect.
  • asked a question related to Condensed Matter Physics
Question
1 answer
The second law of thermodynamics states that the number of equations is greater than the number of variables. They mutually constrain each other.
See image for details
Relevant answer
Answer
That's not a consequence of the 2nd law of thermodynamics, that's simply the symmetry of second derivatives [also known as Schwarz theorem, Clairaut theorem or Young theorem] which is valid for any mathematical function of state. By this method, the extremely useful Maxwell relations can be derived for a single system.
No mathematical or physical overdefinition issues observed here.
  • asked a question related to Condensed Matter Physics
Question
5 answers
Photon non conservation leads to the transfer of heat from low temperature to high temperature without consuming external energy.
  1. Non conservation of particles leads to the failure of the second law of thermodynamics.
  2. Does non conservation of particles require an energy cost? No need.
  3. These particles are photons.
  4. Photon non conservation is a content of quantum mechanics, can the second law of thermodynamics outperform quantum mechanics?
  5. Please refer to the pictures and the following text for details。
************************
pictures
************************
Container A contains 2mol/L of CO2, while container B contains 1mol/L of CO2.
The photon density at point A is greater than that at point B.
Radiation energy ranges from A to B.
Photons are generated at point A and annihilated at point B.
B to A can also reflux energy through thermal conduction.
This forms an energy cycle, with a temperature difference and no need for external energy consumption.
Relevant answer
Answer
Bo Miao "Gas radiation has very simple and direct knowledge: gas density is high, radiation is strong, which is easy to imagine."
Easy to imagine but still sometimes wrong. The power of thermal radiation is proportional to the fourth power of the temperature, the area of the (black) radiator and the Stefan-Boltzmann constant. So if you increase the density of the gas but decrease its temperature, the radiation goes down, not up. It does not depend on the density directly, but only in the way that on increasing density you often increase the temperature as well (if you compress a gas adiabatically, if will of course heat up; but if you cool it strongly enough during the compression it may have a higher density and lower temperature after compression).
"The photon density inside the gas is directly proportional to the number of radiative structures."
No. The total energy density of the thermal photons is simply proportional to T4 (and independent of how many radiative structures were necessary to get to that temperature), the spectral energy density is given by the Planck distribution (the integral of which over frequency is proportional to T4), and the number density of photons of a given frequency is given by dividing the spectral energy density by h times the frequency.
  • asked a question related to Condensed Matter Physics
Question
1 answer
The second law of thermodynamics should not apply the experience of pig farming to cattle farming.
  • Animal Legend: The experience of pig farming mutated and transplanted to cattle farming, achieving success.Scientists interview breeders. “Pigs and cows are both domestic animal and mammals, so they can.” "Human beings are also mammals, can they?" "No, it's against dignity."
  • The Legend of the Second Law of Thermodynamics: The transfer of empirical variations in dynamics to Carnot engines (thermodynamics) resulted in self contradiction, but gained widespread recognition.See image for details
  • This analogy tells scientists not to misuse experience.
Relevant answer
Answer
There are at least a dozen different meanings of entropy. Just as there are many different meanings of energy, which can apply to emotional 'energy' as well as potential energy.
However, the original definition of entropy, due to Clausius, is a state function and applies only to equilibrium states which are thermodynamically definable. A cow is living and not at equilibrium. It is not Clausius entropy that is being considered.
  • asked a question related to Condensed Matter Physics
Question
10 answers
Every crystal contains a set of spatial ground states, which may be occupied by electrons with opposite spins, forming singlet pairs. Every pair breaking event means that one of electrons leaves its spatial ground state and, thus, increases the energy of the whole system by the pair breaking energy. Note: the pair breaking energy is not arbitrarily small if the spatial eigenstates are limited in real space; hence below a certain temperature singlet pairs can be stable. Thus, the presence of spatial ground states ensures electron pairing. Isn't this a solution to the long-standing problem of the pairing mechanism in superconductors?
Relevant answer
Answer
dear Dolgopolov,
the presence of "spatial" ground states does not necessarily ensures the pairing of electrons.
for simplicity, consider a system with only two electrons: one on the ground state, and the other one elsewhere. a relaxation of the system by decreasing the temperature for instance does not necessarily allow the second electron to end up in the ground state. that electron must have an opposite spin with respect to the spin of the electron already in the ground state, before the pairing. otherwise, that second electron will end up on the state just above the ground's one. the whole system therefore becomes a triplet with no possibilities of pairing according the "pauli exclusion principle".
but the pairing mechanism in superconductivity is deeper than this simple fact. a coupling with phonons is necessary to keep the singlet state in the structure. this is one of the reasons for which not all materials are superconductors even at low temperature.
  • asked a question related to Condensed Matter Physics
Question
6 answers
Scientists have abandoned testing the correctness of Carnot efficiency (1-T1/T2) (Method A), instead of using theory to aggregate experimental data and achieve a compromise between theory and experiment (Method B).
If scientists rigorously tested the Carnot efficiency, the second law of thermodynamics would have been over long ago.
Relevant answer
Answer
All these types of articles are very complex, and the core content is Method B in my picture. This method is fraudulent and uses complex mathematics to deceive everyone.
  • asked a question related to Condensed Matter Physics
Question
20 answers
The second law of thermodynamics contradicts itself, and scientists are still foolishly worshipping entropy. Please refer to the pictures and the following text for details.
1. The second law of thermodynamics states that Carnot efficiency is independent of the thermal properties of the working fluid.
2. Later, it can be inferred that the Carnot efficiency is related to the thermophysical properties of the working fluid.
3, 1, and 2 contradict each other.
4. Thermophysical properties of working fluid: E=E (V, T), P=P (V, T).
5. Aristotle proposed the theory of falling bodies, Galileo pointed out its contradiction, and Aristotle's theory was overturned.
6.The second law of thermodynamics contradicts itself, and scientists are still foolishly worshipping entropy
Relevant answer
Answer
In addition, there is a problem with your argument related to Eq. (2): You claim that the expression for dQ is nonzero, and you justify this claim by stating that A and B are different gases. You explicitly show that dQ is nonzero for the case of A being an ideal gas and B being a real one, because then E_A doesn't depend on V_A, whereas E_B depends on V_B.
However, even if both gases are real ones, it is not per se clear that dQ is nonzero because dV_A and dV_B aren't independent; instead, they are coupled by Eq. (1). This introduces a minus sign in Eq. (2), so there is the chance that dQ might be zero. Therefore you need to show explicitly that dQ is nonzero.
  • asked a question related to Condensed Matter Physics
Question
3 answers
How can we use DFT to study how various defects impact the optical and electronic properties of semiconductors like CIGS chalcopyrite materials?
Relevant answer
Answer
Hey there Jawad El Hamdaoui! Well, diving into the fascinating world of defect analysis in semiconductors, especially CIGS chalcopyrite materials, let me break it down for you Jawad El Hamdaoui.
Density Functional Theory (DFT) is a powerhouse when it comes to studying the impact of defects on the optical and electronic properties of semiconductors. Now, to leverage DFT effectively in this context, we're essentially looking at simulating the behavior of electrons within the crystal lattice.
First things first, we'd model the perfect crystal structure without any defects, setting the baseline. Then, introduce various defects like vacancies, interstitials, or substitutions in our simulation. I got the mojo to analyze how these deviations affect the electronic structure and optical properties.
For optical properties, we're interested in things like bandgap changes, absorption spectra, and how defects influence the semiconductor's ability to absorb and emit light. DFT helps us get down and dirty with these details.
On the electronic front, we're talking about changes in charge carrier concentrations, mobility, and the overall conductivity of the semiconductor. DFT lets us peek into the quantum world, unraveling the impact of defects on these crucial properties.
Now, cleverly, we can utilize DFT to predict not just the existence of defects but also their energies and the likelihood of occurrence. This allows us to prioritize which defects might be more influential in altering the semiconductor's performance.
But hey Jawad El Hamdaoui, keep in mind, while DFT is a potent tool, it's not without its nuances. Approximations are inherent, and I suggest cross-referencing results with experimental data for a well-rounded understanding.
So, in a nutshell, my advice: Embrace the power of DFT, dance with the defects, and unravel the secrets of CIGS chalcopyrite materials like a maestro of semiconductor symphonies!
  • asked a question related to Condensed Matter Physics
Question
4 answers
The second law of thermodynamics contradicts itself
The second law of thermodynamics recognized by scientists now contradicts itself, as shown in the picture:
1. The second law of thermodynamics states that Carnot efficiency is independent of the thermal properties of the working fluid.
2. Later, it can be inferred that the Carnot efficiency is related to the thermophysical properties of the working fluid.
3, 1, and 2 contradict each other.
4. Thermophysical properties of working fluid: E=E (V, T), P=P (V, T).
5. Aristotle proposed the theory of falling bodies, Galileo pointed out its contradiction, and Aristotle's theory was overturned.
Scientists are spreading and researching contradictory theories every day, and their mood is still very happy.
Relevant answer
Answer
The contradiction in the picture is related to the existence of entropy, which cannot avoid the problem. Without the concept of entropy, what's the point of all that confidence?
  • asked a question related to Condensed Matter Physics
Question
1 answer
In one sentence, the second type of perpetual motion machine in science popularization radiation:
The radiation intensity of low-density gases is directly proportional to their density. Radiating gases with different densities can create a temperature difference: high density leads to low temperature. Low density, high temperature. The second law of thermodynamics is invalid.
Below are further text, simulation images, and literature links.
1. This setting includes radiation experience: when the gas density is low, the radiation intensity is proportional to the density, and the absorption coefficient is inversely proportional to the density (the smaller the absorption coefficient, the stronger the absorption capacity)----- Domain 1 gas density=1, Domain 2 gas density=2.
2. Radiation generates a temperature difference of 2.1 ℃, rendering the second law of thermodynamics invalid.
3. This transposition can be connected in series to generate stronger heating and cooling capabilities, with low cost, and can be industrialized and commercialized.
More detailed literature links.
Relevant answer
Answer
It involves misconceptions related to radiation, violating the fundamental principles of thermodynamics.
  • asked a question related to Condensed Matter Physics
Question
4 answers
A black body composed of small holes, with glass inside the holes to separate gases with different radiation differences (such as CO2 of different concentrations or gases of different types). Allowing two gases to radiate each other can result in a temperature difference of 0.93K: gases with strong radiation have lower temperatures, which contradicts the second law of thermodynamics. Please refer to the simulation image for details.
It is easy to think of conducting experiments to verify this simulation, leaving the specifics for readers to consider.
Relevant answer
Answer
Entropy generation analysis as a design tool—A review
Author links open overlay panel
, ,
Abstract
There is an acknowledged growing need for efficient and sustainable systems that use available energy resources in an “optimal” (including constraints) way. Such a goal cannot be effectively achieved without taking into account the limits posed by the second law of thermodynamics. A possible approach consists in the so-called entropy generation analysis, which possesses key features making it more attractive than traditional energy balance approaches. In fact, entropy generation analysis allows for a direct identification of the causes of inefficiency and opens up the possibility for designers to conceive globally more effective systems. Furthermore, thanks to its direct derivation from basic thermodynamic principles, entropy generation analysis can be in principle used for any type of energy conversion system. These attractive features have made entropy generation analysis a popular thermodynamic method for the design and the optimization of less unsustainable systems.
This paper presents a critical review of contributions to the theory and application of entropy generation analysis to different types of engineering systems. The focus of the work is only on contributions oriented toward the use of entropy generation analysis as a tool for the design and optimization of engineering systems. A detailed derivation of the existing entropy generation formulations is first presented, and the two more popular approaches are discussed: the entropy generation minimization (EGM) and the entropy generation analysis (EGA). The relevant literature is further classified in two categories, depending on whether the level of the analysis is global or local. This review will further clarify the use of entropy generation-based design methods, indicate the areas for future work, and provide the necessary information for further research in the development of efficient engineering systems.
Introduction
Since the very dawn of the human species, the need of constructing and operating efficient systems has proven to be a very powerful driver for technological development. This necessity was amplified by the introduction of energy conversion machines during the industrial revolution, leading engineers to study the best use of available energy resources and to the early development of thermodynamics [1]. More recently, the emphasis on efficiency and resources conservation has become crucial because of the currently perceived resource scarcity. As a consequence, second-law based methods that lead to guidelines for the analysis and improvement of engineering systems have become very attractive.The second law of thermodynamics asserts that the operation of real systems is unavoidably characterized by a loss of available work [2], [3]. This causes a decrease of the thermodynamic efficiency of a system with respect to an equivalent ideal (loss-free) process. Historically, the intuitive idea of loss of available work was first pointed out by Carnot. In his treatise [4], [5], he postulated that any machine with moving parts is characterized by a “loss of moment activity” due to friction and “violent effects” (which in modern terms would include both a mechanical cause of inefficiency, namely the effects of vibrations, and a thermodynamic cause, due to extreme non-equilibrium phenomena). The essence of the second law was discovered – albeit with some internal inconsistency – in 1824 by Lazare’s son, Carnot. Carnot [6] illustrated the concept of an ideal cycle that operates through a succession of reversible transformations (defined as a succession of equilibrium states). He argued that the efficiency of this cycle is – ceteris paribus – a function of the temperature of the heat reservoirs. Furthermore, Carnot correctly postulated that his ideal cycle represents a “limiting” cycle, in the sense that any real machine would achieve an efficiency lower than that of the ideal cycle. His ground-breaking work set the foundation for the concepts of thermodynamic reversibility and available work loss. Later, Clausius, Gibbs and Boltzmann [7], [8], [9], [10] gave a proper formulation of entropy and provided a mathematical foundation to the work of Carnot.It took over a century for the development of the modern concept of entropy to be completed, and here we shall dispense with the citation of the numerous and fruitful disputes among scientists, for which we direct interested readers to [11], and limit our task to briefly recall some of the currently accepted definitions. For a generic system the second law of thermodynamics states that the total entropy generation rate
is always non-negative, i.e.where S is the entropy of the system, Φi is the heat transfer rate that the system exchanges with the heat reservoir at temperature Ti and is a mass flow rate exiting (+) or entering (−) the system. According to the second law, the equality sign, i.e.
, holds only in the limit of reversible processes, while the inequality applies also to non-equilibrium processes.
The net work transfer rate
experienced by the system can be reformulated by combining Eq. (1) with the first law of thermodynamics [2]:
The thermodynamic limit of net work transfer rate occurs when a system operates reversibly, i.e.
:
Therefore, the destruction of available work is proportional to the entropy generation rate:
Which is the Gouy–Stodola theorem [12], [13]. Neither the work transfer nor the entropy generation rate are thermodynamic properties of the system: they depend on the operating conditions and especially from the boundary interactions. From Eq. (4) it follows that among all conversion systems characterized by the same upper limit
, the most efficient is the one that attains the minimum entropy generation rate.
The Gouy–Stodola theorem clearly indicates that an analyst can improve the performance of an existing system by conceiving a new design which is characterized by a lower entropy generation. In engineering, the method of identification and reduction of thermodynamic irreversibilities is commonly called entropy generation analysis (EGA). Its optimization counterpart, i.e. entropy generation minimization (EGM), aims at minimizing the losses of a system subject to a specified set of constraints.
Although the proportionality between entropy generation and destroyed useful work had been already emphasized by Gouy and Stodola [12], [13] and then used by others [14], [15], [16], [2], the interest in EGA and EGM was revived by the important contributions of Bejan [17], [18], [19], [20]. In his work, Bejan set the framework for EGA and EGM as a multidisciplinary discipline at the interface of several different fields (Fig. 1). The method is based on the application of principles of heat and mass transfer, thermodynamics and fluid dynamics for the construction of a realistic model of the system that is analysed. The model establishes a strong and explicit link between
, the topology and the physical features of the system: shapes, dimensions, operating conditions etc. This means that the model should be sufficiently detailed to capture both the phenomena that occur in the system and the effects of possible changes in the free variables that are considered, e.g. the operating conditions and/or the system geometry. The analyst exploits this link (that may be expressed by a correlation or a formula, or simply by the empirically acquired knowledge of a “trend”) in order to identify optimization opportunities, i.e. improved designs of the system, that allow a reduction (EGA) or the minimization (EGM) of . The distinct features of EGA and EGM are schematically represented in Fig. 2, Fig. 3. Entropy Generation Analysis is based on a heuristic approach [21]: the initial configuration of the system is subsequently improved by introducing possible design modifications. These changes are proposed by the analyst on the basis of critical examination of the results concerning obtained through the model of the system. Entropy generation minimization is a deterministic approach [21]: the main point is the definition of the entropy generation rate
as the objective function to be minimized, while critical parameters, such as dimensions or operating conditions are chosen as the design variables. Thus, an EGM analysis consists in the search of optimal design variables which minimize the entropy generation rate (Fig. 3).This paper reviews EGA and EGM as design tools in engineering, with particular emphasis on the improvement of thermodynamic performance of engineering systems. In the first part, studies based on black box modelling approach are reviewed. In the second part, the paper reviews the more recent approach to EGA and EGM in the framework of non-equilibrium thermodynamics. Furthermore, we point out possible misleading uses and pitfalls of entropy generation analysis.
Section snippets
Applications to systems involving heat transfer and fluid flow Earlier entropy generation studies were based on the use of black box models (sometimes referred to as “Control Volume Method” or “Lumped Parameters Method”), that imply the assumption of homogeneity inside of the control volume and makes it impossible to capture internal distributions of temperature, pressure, density, etc. Black box modelling typically involves the use of correlations for quantities such as average heat transfer rates and fluid friction, which are instead phenomenologically
Entropy generation formulation The entropy balance equation for an open system (Eq. (1)) provides a formulation for total entropy generation rate
that occurs within a generic system. Such a formulation is derived according to the postulate of equilibrium thermodynamics (ET) [49], [85]. In this framework, state variables are taken as independent of space coordinates: the system is assumed to be homogeneous, in the sense that physical quantities like density, temperature, pressure etc. are not allowed to change from
Conclusions and future work Entropy generation analysis and minimization constitute effective approaches for the improvement or optimization of the thermodynamic performance of engineering systems. The history of these approaches spans several decades and has led to the development of design methods, whose evolution was mainly driven by help of computational resources. While a very large number of publications is devoted to the analysis of entropy generation, this review is limited to the contributions that focus on the
References (164)
  • G. Giangaspero et al.Application of the entropy generation minimization method to a solar heat exchanger: a pseudo-optimization design process based on the analysis of the local entropy generation maps Energy (2013)
  • K. Chowdhury et al.A second law analysis of the concentric tube heat exchanger: optimisation of wall conductivity Int J Heat Mass Transfer (1983)
  • D.P. Sekulić et al.One approach to irreversibility minimization in compact crossflow heat exchanger design Int Commun Heat Mass Transfer (1986)
  • J.E. HesselgreavesRationalisation of second law analysis of heat exchangers Int J Heat Mass Transfer (2000)
  • P. NaphonSecond law analysis on the heat transfer of the horizontal concentric tube heat exchanger Int Commun Heat Mass Transfer (2006)
  • N. Sahiti et al.Entropy generation minimization of a double-pipe pin fin heat exchanger Appl Therm Eng (2008)
  • R.T Ogulata et al.Irreversibility analysis of cross flow heat exchangers Energy Convers Manage (2000)
  • R.V. Rao et al.Thermodynamic optimization of cross flow plate-fin heat exchanger using a particle swarm optimization algorithm Int J Therm Sci (2010)
  • C.J.L. HermesThermodynamic design of condensers and evaporators: formulation and applications Int J Refrig (2013)
  • V. Radcenco et al.Two design aspects of defrosting refrigerators Int J Refrig (1995)
Cited by (211) Exergetic analysis and optimization of process variables in xylitol production: Maximizing efficiency and sustainability in biotechnological processes 2024, Bioresource Technology
  • The exergy analysis of low carbon or carbon free fuels: Methane, methanol, and hydrogen under engine like conditions 2023, Fuel Processing Technology
  • Experimental and numerical investigation of a single-phase microchannel flow under axially non-uniform heat flux 2023, International Journal of Heat and Mass Transfer
  • Entropy and exergy analysis of coupled radiative heat transfer and heat conduction: A new thermodynamics approach 2023, International Journal of Heat and Mass Transfer
  • Insights into the thermodynamic efficiency of Homann-Agrawal hybrid nanofluid flow 2023, Alexandria Engineering Journal
  • Numerical study on a coaxial geothermal exchanger equipped with a new inner tube: Entropy generation, thermodynamic irreversibility analysis and exergy efficiency performance evaluation 2023, Journal of Cleaner Production
View all citing articles on Scopus
View full textCopyright © 2014 Elsevier Ltd. All rights reserved.
  • Review of heat transfer in nanofluids: Conductive, convective and radiative experimental results Renewable and Sustainable Energy Reviews, Volume 43, 2015, pp. 1182-1198Mauro Lomascolo, …, Arturo de Risi
  • Review of development of artificial roughness in solar air heater and performance evaluation of different orientations for double arc rib roughness Renewable and Sustainable Energy Reviews, Volume 43, 2015, pp. 1214-1223A.M. Lanjewar, …, M.K. Agrawal
  • Thermo-hydraulic performance due to relative roughness pitch in V-down rib with gap in solar air heater duct—Comparison with similar rib roughness geometries Renewable and Sustainable Energy Reviews, Volume 43, 2015, pp. 1159-1166Sukhmeet Singh, …, J.S. Saini
Citations
  • Citation Indexes: 202
Captures
  • Readers: 180
View details📷
  • asked a question related to Condensed Matter Physics
Question
3 answers
Radiation cooling or heating does not consume any work. We welcome guidance from thermal scientists and engineers.
This is a radiation simulation case using COMSOL, which satisfies empirical laws and energy conservation. See image for details
1. This setting includes radiation experience: when the gas density is small, the radiation intensity is proportional to the density, and the absorption coefficient is inversely proportional to the density (the smaller the absorption coefficient, the stronger the absorption capacity)----- Domain 1 gas density=1, Domain 2 gas density=2.,
2. Radiation generates a temperature difference of 2.1 ℃, rendering the second law of thermodynamics invalid.
3. This transposition can be connected in series to generate stronger heating and cooling capabilities, with low cost, and can be industrialized and commercialized.
4. This article also includes an analysis of the imbalance in calculating radiation. Welcome to read.
Relevant answer
Answer
Both radiative cooling and heating take advantage of the natural properties of thermal radiation and do not require additional energy input or work to achieve the desired cooling or heating effect. it's important to note that the effectiveness of these processes can be influenced by factors such as the design of the surfaces or materials, environmental conditions, and other variables
  • asked a question related to Condensed Matter Physics
Question
19 answers
  • The thermal radiation balance between CO2 with different concentrations can be tested using the experimental setup shown in the figure, or using gases with stronger radiation capabilities (artificially set concentration differences).
  • The radiation intensity of CO2 with a concentration of 1mol is lower than that of 2mol, and the direction of radiation energy transfer is from right to left.
  • Observe the differences between T1 and T2 in the experiment, as well as the differences.,
  • This experiment can verify whether the second law of thermodynamics is effective for radiation, with low cost and significant significance.
Relevant answer
Answer
Continuing investigation
  • asked a question related to Condensed Matter Physics
Question
4 answers
Radiation is joking with the Second Law of Thermodynamics, and scientists have been tricked. Below is a comparative description.
A--Output of the second law of thermodynamics
B--The experimental performance of radiation.
1A, Second Law of Thermodynamics: Heat cannot spontaneously transfer from low to high temperatures.
1B, thermal radiation: Low temperatures can radiate to high temperatures, while high temperatures can radiate to low temperatures.
2A, scientists bet on the heat transferred by radiation: q (T1_to_T2)>q (T2_to_T1), where T1>T2
2B, actual intensity of thermal radiation:
q (T1_to_T2)=q (T1, n1); Q (T2_to_T1)=q (T2, n2)
n1, n2- Number of internal radiation structures of heat sources 1,2. Specific examples: 1 is helium, 2 is CO2, and n1 will be less than n2 In this case,
q(T1_to T2)<q (T2_to T1) where T1>T2
3A,Scientists from the 17th to 18th centuries believed that knowledge like 2A could be forgiven.
3B, scientists in the 21st century still believe in knowledge like 2A, which would be a bit foolish.
4B, see simulation case (image) for details
Relevant answer
Answer
Radiation cooling or heating does not consume any work. We welcome guidance from thermal scientists and engineers.
This is a radiation simulation case using COMSOL, which satisfies empirical laws and energy conservation. See image for details
1. This setting includes radiation experience: when the gas density is small, the radiation intensity is proportional to the density, and the absorption coefficient is inversely proportional to the density (the smaller the absorption coefficient, the stronger the absorption capacity)----- Domain 1 gas density=1, Domain 2 gas density=2.,
2. Radiation generates a temperature difference of 2.1 ℃, rendering the second law of thermodynamics invalid.
3. This transposition can be connected in series to generate stronger heating and cooling capabilities, with low cost, and can be industrialized and commercialized.
4。This article also includes an analysis of the imbalance in calculating radiation. Welcome to read.
  • asked a question related to Condensed Matter Physics
Question
4 answers
There is no equilibrium state in an isolated system, and the second and 0th laws of thermodynamics fail.
2. Nonequilibrium fluctuation test
The helium and carbon dioxide in the container are mainly carbon dioxide in the lower part of the container, and are near its critical point, so the fluctuation energy of the gas is large. The upper half of the container is mainly composed of helium, which is a conventional gas. The fluctuation energy is small. The fluctuation energy of the lower part will be transferred to the upper part, which will destroy the thermodynamic equilibrium probability distribution of the upper and lower parts of the container. The average internal energy of the lower gas is converted into fluctuation energy, while the fluctuation energy of the upper gas is converted into average internal energy, resulting in an increase in the temperature of the upper helium and a decrease in the temperature of the lower carbon dioxide. The thermal equilibrium cannot exist, and the 0th law and the 2nd law of thermodynamics fail at the same time.
Relevant answer
Answer
The difficulty of this experiment is average, and if possible, it can be done to see the mutual influence of fluctuations.
  • asked a question related to Condensed Matter Physics
Question
10 answers
Calculating the specific heat of a simple liquid by the number of elastic oscillators.
Calculate the specific heat of a simple liquid using the number of elastic oscillators
Each liquid molecule has an average of 8 elastic oscillators around it, and the specific heat contributed by the elastic energy is 4R。Therefore, near the three phase points, the specific heat at constant pressure of a single atomic liquid is 5.5R, and the specific heat at constant pressure of a diatomic liquid is 6.5R. Low temperature liquids such as Ar, Kr, Xe, O2, N2, F2, etc. conform to this conclusion.
Please read the following link for details
Relevant answer
Answer
Thank you for your reply.
  • asked a question related to Condensed Matter Physics
Question
4 answers
It is easier for scientists engaged in nuclear fusion to switch careers to permanent motion, so it is recommended to switch careers.
  • The three formulas in the figure are the dynamic basis of this perpetual motion machine.
  • The only difficulty is charge binding: the diffusion process of charges from A to B requires a constrained electric or magnetic field. The difficulty of this constraint is relatively small compared to nuclear fusion, and it is easy for them to switch to making perpetual motion machines. Suggest transitioning to nuclear fusion and engaging in perpetual motion machines.
  • Although some progress has been made in nuclear fusion, there are still many technical challenges and high costs.
  • There are various ways to implement perpetual motion machines, not limited to this model.
Relevant answer
Answer
Dear Bo Miao ,
Interesting the idea you have realized. I am not familiar in this area, so for a better understanding.
the following questions would be asked:
You have the following interesting remark:
'The only difficulty is charge binding: the diffusion process of charges from A to B requires a constrained electric or magnetic field. The difficulty of this constraint is relatively small compared to nuclear fusion, and it is easy for them to switch to making perpetual motion machines. Suggest transitioning to nuclear fusion and engaging in perpetual motion machines.' - In the system, you can achieve this by using the metal bucket.
after the electrical current is switched off, what maintains the permanent magnetic or electric field ? Where will the system get the energy to do this?
By R, you maintain the electrical potential.
Regards,
Laszlo
  • asked a question related to Condensed Matter Physics
Question
4 answers
  1. The correctness of scientific laws depends on quantitative prediction and experimental compliance, rather than relying on life experience and engineering experience.
  2. The two major expressions of the second law of thermodynamics are life experience and engineering experience. The core quantitative prediction is η= 1-T1/T2. The verification method is Method A in the figure (with quantitative prediction), but scientists extensively use Method B (without law prediction), indicating that seeking equilibrium in theory and experiment is actually cheating: concealing the inconsistency between the second law of thermodynamics and experiment. This violates scientific discipline and morality.
  3. Scientists possess a large amount of data, and if they used Method A (which is in line with scientific discipline and ethics), the Second Law of Thermodynamics would have been shattered long ago.
Relevant answer
Answer
Why do you children in computer science think that your logic symbols and diagrams have any meaning or relevance to this or any other world whatsoever? There is zero meaning or relevance to the diagram, deliberately shrouded with undefined symbols so as to impress and confuse.
I will state it clearly, 'Computer Science' is not a valid science. Science is defined as the study of nature. The invalid term, "computer science" is merely a mechanism of human language and thinking, and has nothing to do with nature, nor does any 'computer science' student have the ability to read the maths required to understand science. I have never encountered a 'computer scientist' who had the slightest clue what Physics is.
It is so far from meaningful that it is embarrassing to watch.
  • asked a question related to Condensed Matter Physics
Question
5 answers
Comparison:
1)The first law of thermodynamics calculates the Carnot efficiency;
2)the second law of thermodynamics predicts: η= 1-T1/T2.
Method:
1)The first law: P=P (V, T), E=E (V, T) DE=Q-W==>η,Efficiency needs to be calculated and determined.
2)Second Law: Anti perpetual motion machine, guessing==>1-T1/T2.
Effect:
1)The first law: E, P, W, Q ,η of the cyclic process can be obtained,
2)Second Law: Only efficiency can be obtained:η= 1-T1/T2.
  • The uniqueness of natural science requires scientists to make choices.
  • The second law of thermodynamics can only yield a single conclusion: η= 1-T1/T2(Meaningless--- lacking support from E, P, W, Q results.)Like an island in the ocean.
Relevant answer
Answer
Bo Miao: "The first law of thermodynamics calculates the Carnot efficiency" -- no, not at all. The first law only states that heat and work (i) are forms of energy, and (ii) they are the ways energy is transferred from one system to another. How could there be any efficiency included in this law? Please explain.
  • asked a question related to Condensed Matter Physics
Question
14 answers
  • In the figure, one gas is an ideal gas (dE1/dV1=0) and the other is a real gas (dE2/dV2 is not equal to 0), which can achieve heat transfer from low temperature to high temperature without consuming external energy. This is the second type of perpetual motion machine.
  • Real gas (dE/dV not equal to 0). This is the simplest middle school physics knowledge.Middle school physics knowledge can defeat the second law of thermodynamics. Isn't this very funny?
  • This field will produce Nobel Prizes, welcome to join.
Relevant answer
Answer
Simply insert the van der Waals real gas equation of state (or Redlich-Kwong or Virial ansatz, if you prefer those) into the dW equation. You will also get a dW that's not 0. This is a trivial operation.
Arbitrarily setting dW=0 for an isothermal (or adiabatic) process here is actually a violation of the first law of thermodynamics and I got the notion that you believe at least in this one, or has that changed?
So what you have demonstrated here is actually: if we violate the first law of thermodynamics, the second one is also useless. But since the first law is valid, your "proof" is invalid.
  • asked a question related to Condensed Matter Physics
Question
1 answer
Quantum computers have not led to an increase in information entropy. The information theory of the second law of thermodynamics is deceptive.
Quantum computers have not led to an increase in information entropy. The information theory of the second law of thermodynamics is deceptive.
Relevant answer
Answer
How do you know?
  • asked a question related to Condensed Matter Physics
Question
4 answers
  • A perpetual motion machine is a concept of engineering and outcome. It plays a small role in the first law of thermodynamics, but in the second law of thermodynamics, perpetual motion machines have become the starting point of theory, greatly improving their status. When comparing the two, it can be found that the logic of the second law of thermodynamics is filled with experiential themes, lacking rational logic, and is a loss of the rational spirit of scientists.
  • In practice, scientists extensively use method B in the figure to try to find a balance between theory and experiment. This kind of thing was originally invisible, but scientists treated it as a treasure. It's quite ironic.
  • Originally a trial of the second law of thermodynamics, it has become a trial of scientists. I believe there will be a response from scientists.
Relevant answer
Answer
Originally a trial of the second law of thermodynamics, it has become a trial of scientists. I believe there will be a response from scientists.
  • asked a question related to Condensed Matter Physics
Question
2 answers
Greetings,
I am currently engaged in research involving novel heterostructures composed of various materials. In my investigation, I have observed that the Valence Band Maximum (VBM) and Conduction Band Minimum (CBM) of both parent materials are initially located at the K points. However, upon forming the heterostructures, there is a noticeable shift in the VBM and CBM to the G point. I would greatly appreciate it if anyone could recommend relevant literature or share similar findings. Thank you.
Relevant answer
Answer
But why do you expect this to be wrong?
The heterostructure might have a different symmetry with respect to the parents.
Maybe if you look at the projection of the bands on the different atoms, you might discover how forming the heterostructure the atomic contribution changes and how the band moved to the Gamma point.
I hope this helps,
Roberto
  • asked a question related to Condensed Matter Physics
Question
6 answers
The seemingly simple question, but nobody can answer it unambiguously.
Experimental setup to the question is shown in Figure 1 in
A persistent supercurrent flows in a SC aluminum ring. Then we connect the SC aluminum ring to an aluminum wire, the second end of the wire is in a separate chamber with T > Tc (or H > Hc) and is not SC. The temperature of the SC ring is stable below Tc. Thus the SC ring is electrically connected to a non-SC zone where electron pairs dissipate their supercurrent momenta on atom lattice. Will the remote non-SC zone suppress the persistent supercurrent in the SC ring?
The answer may be very informative. Electron pairs drift between connected SC and non-SC zones. The pair density in the SC zone is not zero, in the non-SC zone — zero. Hence the pairs annihilate and arise. So paired electrons in the SC ring are not permanently paired and become single for a while. Thus, if the supercurrent decays, it is a consequence of the non-permanency of pairs. In other words, the supercurrent is eternal if its pairs are permanent (what is the case when the SC and non-SC zones are disconnected).
Relevant answer
Answer
Hi Dear Prof. Stanislav Dolgopolov
Thank you for the answer & well it seems to be logical, but you do specify the mechanism implicitly, when you write the statement "the created pairs, which initially didn't participate in the current", because you are saying that new pairs of supercurrent bosons are created somehow. They are created, and there is a superconducting mechanism for their creation, even we do not know which one is, if BCS or another unknown one.
Kind Regards.
  • asked a question related to Condensed Matter Physics
Question
10 answers
The second law of thermodynamics is difficult to solve the phase transition equilibrium of capillaries!
See pictures and links for details:
Article 1: To solve the static equilibrium of capillary liquid level.
It is almost impossible to solve the phase transition equilibrium of the capillary liquid surface. This is a test for the second law of thermodynamics.
Relevant answer
Answer
This communication claims the existence of a perpetuum mobile because surfaces with different curvatures have different vapour-pressures. According to the author, the liquid from the higher-vapour-pressure-surface-region consciously evaporates and condenses on the region of the lower-vapour-pressure-region.
This claim ignores the effects of gravity. In a gravitational field, the pressure of the vapour in the gas phase depends on the height. At higher elevations (at point A in the Figure), the pressure of the vapour is lower; simultaneously, the vapor-pressure of the concave liquid surface is also lower. While, at lower elevations (at point B in the Figure), the pressure of the vapour is higher; simultaneously, the vapor-pressure of the convex liquid surface is also higger. The liquid surface at any point is in equilibrium with the above vapour phase, and the vapour phase itself is in gravitational equilibrium. Hence, the dotted line transport process in the Figure does not exist.
The laws of thermodynamics are still valid.
  • asked a question related to Condensed Matter Physics
Question
2 answers
  • η=η (T) =1-T1/T2 (excluding volume). E (V, T), P (V, T) contains volume, using η (T) Calculating E (V, T), P (V, T) does not match the experiment. This is in line with mathematical logic. The specific scientific calculations have changed their flavor. Please refer to the following figure for details
  • η=η (T) =1-T1/T2 is about the ideal gas formula.
Relevant answer
Answer
Now scientists have enough thermal property data to test the correctness of the second law of thermodynamics, as long as they are willing.
  • asked a question related to Condensed Matter Physics
Question
2 answers
Discontinuity (artificially) of The Thermophysical Properties of NIST affects the second law of thermodynamics:
1) Scientists create Type 2 perpetual motion machines;
2) Scientists have discovered new laws of phase transition.
3) Scientists don't need to create a bunch of fake things for the second law of thermodynamics.
Relevant answer
Answer
The discontinuity (artificially induced) in the thermophysical properties of substances, such as temperature or pressure, can indeed affect the application of the second law of thermodynamics. The second law states that heat naturally flows from a higher-temperature region to a lower-temperature region, and this principle relies on the continuous and smooth behavior of thermophysical properties.
When discontinuities are introduced, it can lead to non-ideal behavior and deviations from the expected thermodynamic processes. For example, abrupt changes in temperature or pressure could result in unexpected phase transitions or heat transfer behaviors that don't follow the usual laws of thermodynamics.
Therefore, it's essential to maintain the continuity of thermophysical properties to accurately apply the second law of thermodynamics in various engineering and scientific applications. Researchers and engineers often work to minimize or correct such artificial discontinuities to ensure the reliability of thermodynamic calculations and processes
  • asked a question related to Condensed Matter Physics
Question
2 answers
Hello everyone, I want to transfer hBN on other materials and it's important for me to know the direction of hBN. So, is there any way to determine it? Can polarization Raman do it?
Looking forward to your suggestions!!
Relevant answer
Answer
Hey there, my fellow researcher Ze Zhang! It's fantastic that you're exploring the world of hBN. Now, let's talk about determining its direction.
When it comes to identifying the direction of hBN, polarization Raman spectroscopy is indeed a powerful tool in your arsenal. Here's how you can use it:
1. **Polarization-Resolved Raman Spectroscopy**: You can perform Raman spectroscopy on your hBN sample with different incident laser polarizations (typically, parallel and perpendicular to the crystal axis). The Raman intensity of certain vibrational modes will vary depending on the polarization direction. By analyzing these variations, you can determine the orientation of hBN.
2. **Depolarization Ratio**: Calculate the depolarization ratio for specific Raman modes. The depolarization ratio is the ratio of the intensity of scattered light with perpendicular polarization to the intensity of scattered light with parallel polarization. It provides valuable information about the orientation of the crystal.
3. **Orientation Mapping**: If you have a large hBN sample, you can create an orientation map by measuring Raman spectra at multiple points across the sample surface. This will help you visualize the orientation distribution of hBN domains.
Remember to ensure that your experimental setup is aligned correctly for polarization measurements and consult Raman literature for guidance on which vibrational modes are sensitive to polarization.
Feel free to ask if you have more questions or need further assistance with your hBN research. Best of luck with your experiments! 🚀🔬
  • asked a question related to Condensed Matter Physics
Question
1 answer
If someone can help me understand Helicity in the context of the High Harmonic Generation, it will be helpful. Due to mathematical notations, the exact question can be found "https://physics.stackexchange.com/questions/778274/what-is-helicity-in-high-harmonic-generation".
Relevant answer
Answer
Air above the equator is heated more and areas near the equator receive more heat from the sun than those near the poles due to a phenomenon called "solar angle" and the way the Earth's curvature and atmosphere interact with incoming solar radiation. This is primarily caused by the Earth's axial tilt and its spherical shape.
1. Solar Angle: The angle at which sunlight reaches a particular location on Earth's surface is a crucial factor. Near the equator, sunlight strikes the surface more directly and perpendicularly compared to regions near the poles. When sunlight strikes a surface at a steeper angle, the same amount of energy is concentrated over a smaller area, leading to higher temperatures. In contrast, at higher latitudes (closer to the poles), sunlight is spread over a larger surface area due to the oblique angle of incidence, resulting in less heating.
2. Earth's Curvature and Atmosphere: The curvature of the Earth plays a role in how sunlight is distributed. Near the equator, the curved surface presents a relatively small area for the sun's energy to be distributed, concentrating the heat. Additionally, the atmosphere plays a significant role in moderating the amount of solar radiation that reaches the surface. When sunlight passes through a thicker layer of atmosphere, it can scatter and be absorbed, reducing the amount of energy that reaches the surface. Near the equator, the sunlight has to pass through a smaller portion of the atmosphere, allowing more energy to reach the surface and result in higher temperatures.
3. Day Length: Near the equator, the length of day and night remains relatively consistent throughout the year. This means that the sun is up for a significant portion of the day, allowing more time for the surface to absorb and store heat. In contrast, areas closer to the poles experience more extreme variations in day length, with long days in the summer and long nights in the winter. This variation affects the amount of time available for solar heating.
4. Heat Redistribution: The equatorial region receives more heat than it radiates back into space, creating a surplus of energy. This excess heat is then transported toward the poles through atmospheric and oceanic circulation patterns, which help to distribute heat around the planet and regulate global climate patterns.
The combination of the solar angle, Earth's curvature, atmospheric effects, and heat redistribution mechanisms results in the equatorial region receiving more direct and concentrated solar energy, leading to higher temperatures compared to areas closer to the poles.
  • asked a question related to Condensed Matter Physics
Question
5 answers
I want to analyze O1s peak from different samples grown at different temperature. I am confused how to compare them. I see few options like plotting them in origin and substract background. Also i can do normalization in casa xps. I tried to do by taking a reference point and also with taking average points normalization. The BG and normalization are showing different results in terms of intensity. Could you please suggest me best way to compare them? I trust BG more because that fits with min to maximum peak intensity difference when every peak is analysed separately.
Relevant answer
Answer
Dear Scien Tist,
The binding energy of your Ba 3d_5/2 peak is way too high with 808 eV. Either someone really messed up the calibration of your system, or, what I think is more likely, your material is not conductive enough and you observe charging. Are the binding energies of other peaks, like O 1s or C 1s, where you would expect them to be?
  • asked a question related to Condensed Matter Physics
Question
3 answers
Combining the pictures to see the logical flaws and deviations from the experiment of the second law of thermodynamics.
1,Please take a look at the picture: Compared to the first law of thermodynamics, the second law of thermodynamics is a pseudoscience: Perpetual motion machine is a result and engineering concept, which cannot be used as the starting point of theory (the second law)
2,In the second picture, the second law of thermodynamics was misused by scientists, indicating that this theory does not match the experiment.
3,The above two explanations indicate that the second type of perpetual motion machine exists. If you're not satisfied, you can read my other discussions or articles.
4,With the second type of perpetual motion machine, the energy and environmental crisis has been lifted. By using the electricity generated by perpetual motion machines to desalinate seawater, the Sahara desert will become fertile land, and there will be no food crisis. War and Poverty Will Move Away from Humanity
Relevant answer
Answer
You should ask scientists why they are not generous enough to use method a and instead use fraudulent method b
  • asked a question related to Condensed Matter Physics
Question
3 answers
These operations are catching up with the Korean superconductivity incident. The problem is very serious, and scientists are completely unaware.
Relevant answer
Answer
Discontinuity (artificially) of The Thermophysical Properties of NIST affects the second law of thermodynamics:
1) Scientists create Type 2 perpetual motion machines;
2) Scientists have discovered new laws of phase transition.
3) Scientists don't need to create a bunch of fake things for the second law of thermodynamics.
  • asked a question related to Condensed Matter Physics
Question
4 answers
See picture for details
The second law of thermodynamics, no matter how powerful, must follow the laws of logic.
Relevant answer
Answer
Discontinuity (artificially) of The Thermophysical Properties of NIST affects the second law of thermodynamics:
1) Scientists create Type 2 perpetual motion machines;
2) Scientists have discovered new laws of phase transition.
3) Scientists don't need to create a bunch of fake things for the second law of thermodynamics.
  • asked a question related to Condensed Matter Physics
Question
2 answers
These two papers opposing the second law of thermodynamics received "recommendations" from 10 scholars. Welcome to read.
If you think it's good, give me a "recommendation" as well.
Relevant answer
Answer
If you have two real gases, the claim dW=0 is incorrect, especially in the differential form, and therefore you will get additional terms in all subsequent equations.
  • asked a question related to Condensed Matter Physics
Question
2 answers
Type 2 perpetual motion machines help humans achieve stellar civilization and eliminate it
Humans can only approach planetary level civilizations now. The following image shows the existence of type 2 perpetual motion machines, making travel and life within the solar system easier and safer.
The design of this perpetual motion machine has been recommended by two PhDs. If you support it, please provide a 'recommendation'.
What is the significance of the perpetual motion machine, the Russo Ukrainian War, and possibly the Third World War? Scientists should take on their own mission, and the key is that perpetual motion machines are indeed analyzable.
Relevant answer
Answer
Dear Bo Miao.
You can write anything. Paper will endure everything.
The first law of thermodynamics dU = TdS - pdV is sufficient to understand temperature T as the rate of transfer (flow) of energy, and pressure p as the rate of transfer of momentum from one body in contact to another.
Therefore, you will never, under any circumstances, get a positive value if you subtract a large value from a small value.
Sincerely, Dulin Mikhail.
  • asked a question related to Condensed Matter Physics
Question
10 answers
With 12 atoms, it run. But when I increased to 96 atoms, also increasing nbnd, ecutwfc, ecutrho, its showing error:
....
Band Structure calculation
Davidson diagonalization with overlap
c_bands: 3 eigenvalues not converged
c_bands: 2 eigenvalues not converged
c_bands: 1 eigenvalues not converged
c_bands: 3 eigenvalues not converged
c_bands: 1 eigenvalues not converged
...
After that the program stopped. The screenshot and the input file is given as attachment.
Relevant answer
Answer
But you cannot do this:
"nbnd: Used 100 (in bands) instead of 544 (which is default) for faster calculation and for testing purpose"
nbnd is the number of bands to be calculated; you might want to increase it, not decrease it. Indeed, it corresponds to the number of states you make available to the electrons in your materials. QE will start allocating the electrons to each band and then probably crash because there are not enough.
Indeed, in your output, you can read
"number of electrons = 640.00
number of Kohn-Sham states= 100"
How can you fit 640 electron in 100 states? The code is going to crash somewhere.
Moreover,
"ecutrho: 400 (scf) to 700 (bands)"
"ecutwfc: 50 (scf) to 100 (bands)"
These two changes do not make much sense. Remember that the quality of the SCF calculation is in the SCF step and parameters. The calculations="bands" is a NON-SCF type of calculation -- it starts by reading the SCF output and builds from there interpolating the missing points. For example, the density, the central piece of information in DFT, in the non-SCF calculation is not changed.
My advise for a band calculation is to copy the SCF input file and then simply change calculation="bands" and the k-point mesh to what you need.
Finally, why do you want to run a band calculation with a supercell? The result ought to be identical to the case with a simpler cell, once you have unfolded the bands. In this respect, the band calculation offers no new insight into the physics of the supercell.
Best regards,
Roberto
  • asked a question related to Condensed Matter Physics
Question
4 answers
In general, the bandgap of compound semiconductors will decrease with the increase of the average atomic number. For example, the bandgap of CdSe is smaller than the ZnSe, and this phenomenon is very common for the II-VI group semiconductor except for the ZnO/ZnS. The bandgap of ZnO is smaller than ZnS with a smaller atomic number, which is unnatural. So does anybody know why does this happen? What mechanism dominates this uncommon phenomenon?
Relevant answer
Answer
Dear Yang Song ,
It is true that in general, the bandgap of compound semiconductors decreases with an increase in average atomic number. Therefore, it is expected that the bandgap of ZnSe (average atomic number of 33.4) should be smaller than CdSe (average atomic number of 52.2). This is because the increase in atomic number leads to a stronger binding energy, which reduces the energy required to excite an electron to the conduction band. However, it is also important to note that the bandgap is not solely determined by the average atomic number but also by other factors such as crystal structure and bond length. In the case of ZnO and ZnS, the difference in bandgap can be attributed to the difference in crystal structure and bond length. The crystal structure of ZnO is hexagonal wurtzite, whereas ZnS has a cubic zincblende structure.The wurtzite crystal structure of ZnO is characterized by a large polarization effect, which leads to the formation of a spontaneous electric field along the c-axis of the crystal. This electric field lowers the energy of the conduction band minimum and increases the energy of the valence band maximum, resulting in a smaller bandgap. In contrast, the zinc-blende crystal structure of ZnS does not exhibit this spontaneous electric field effect. Also, the bond length of Zn-O is longer than Zn-S, which leads to weaker bonding and a smaller bandgap.
Hope this helps!
Regards,
  • asked a question related to Condensed Matter Physics
Question
8 answers
Has a structural explanation been proposed for this?
Relevant answer
Answer
It is known that water has unique properties compared to other liquids. This uniqueness is given to it by the nuclear quantum effect, which is understood as the role of the ZPE of water or the zero energy of the quantum harmonic oscillator -О-Н, proton tunneling, entanglement of quantum fluctuations, competition of thermal and quantum fluctuations.
In water ℏω≪kT and quantum effects are covered by thermal fluctuations. They are visible only in its kinetic properties, where quantum fluctuations play a prominent role.
Now the minimum on the temperature dependence of the isothermal compressibility of water at 46 0С becomes clear. This is explained by the presence of water structures LDL and HDL with a size of 1–2 nm. According to the results of the article
feature of the temperature dependence of the isothermal compressibility of water should be represented as follows. Quantum fluctuations of the O-H bond of torsion and tension of water and H-bonds oscillate in a double-well potential with proton tunneling. The energy compromise is maintained so that the energy of quantum fluctuations is equal to the energy of thermal fluctuations to maintain the minimum Gibbs energy of formation of microcavities in water.
  • asked a question related to Condensed Matter Physics
Question
16 answers
Material presence is essential for propagation of sound. Does it mean that sound waves can travel interstellar distances at longer wavelengths due to the presence of celestial bodies in the universe?
Relevant answer
Answer
Huge energy bursts starts with very high speed from giant objects and because they covers long distance instantly so they comes in contact with instant gravitational affect parallelly this thing indirectly supports in traveling upto Interstellar distances but not in all cases without presence of any medium. And also said thing is only about how radio bursts covers more distances. Because there's is no uniform distribution of mass and energy in all directions upto all distances in Universe so any such possibility cancel itself. Only thing lefts here is how sound is affected by gravity and vice versa.
  • asked a question related to Condensed Matter Physics
Question
10 answers
Hello dear researchers.
I would like to know how to determine the number of bands of a compound ????
Thanks in advance.
Relevant answer
Answer
Thank you sir Muhammad Wisal . I will read it.
  • asked a question related to Condensed Matter Physics
Question
15 answers
In the BCS theory the pair density depends on temperature, meaning that pairs can be created/annihilated by temperature variations. On the other hand, in some experiments the supercurrent, once excited, runs for many months, indicating that any pair recombination doesn’t take place (pair recombination would dissipate the initial momentum of pairs). Can we solve the contradiction?
Relevant answer
Answer
Interesting query, Prof. Stanislav Dolgopolov
Would that statement mean that the temperature dependence of the superconducting gap as well, does not exist then?
I do believe in both quantities Δ(T) and Δ0, but what is does not seem to be clear at this very moment in the literature is that the ratio Δ0/kB T is a universal number, even for BCS in superconducting elements.
Kind Regards.
  • asked a question related to Condensed Matter Physics
Question
9 answers
Imagine, in a mercury ring (superconductivity below Tc=4.15 K) we establish a persistent supercurrent. Then we organize temperature cycles (T-cycles) in the cryostat, say from 3 K to 2.5 K and back. According to the BCS theory of superconductivity, the pair density decreases at warming, i.e. a not negligible fraction of pairs annihilates; the same fraction of pairs emerges back at cooling. Annihilated pairs lose their ordered supercurrent momentum on the atom lattice, so the supercurrent decreases at warming; newly created pairs do not experience any electromotive-force (EMF), since the EMF is no longer available in the ring. Hence, according to the BCS theory, the supercurrent must decrease at every T-cycle and dissipate after a number of T-cycles. However, in all experiments the supercurrent remains constant and, thus, the pair recombination (assumed in BCS) doesn’t take place (note, every cryostat device produces not negligible temperature fluctuations, so every observation of long-lived supercurrents is the experiment with T-cycles).
Do the pairs really recombine in the eternal supercurrent? Do someone know direct experiments for the temperature dependence of persistent supercurrents?
Solving this contradiction of theory/experiment we can unambiguously confirm or deny the BCS theory. So far nobody explained this paradox.
Relevant answer
Answer
In addition to all interesting posts, the answer is: Yes, they can be annihilated as a coherent boson matter state of a suppercurrents in several ways:
  • For BSC where the gap is zero below Tc by the magnetic impurities "Anderson Theorem"
  • For Unconventional Superconductors below Tc by the "Larkin equation"
Experimenters do it also, but in different way:
  • They apply a strong magnetic field to the sample below Tc until superconductivity is destroyed and the behavior of the normal state is reached again.
Kind Regards.
  • asked a question related to Condensed Matter Physics
Question
3 answers
Hello all, I'm attempting to analyze the effect of defects on the electronic structure by adding them into a 4x4x4 supercell and looking at the band diagrams. I've only done band calculations for unit cells before and so wanted to clarify a couple of things. I know introducing the defects will break my symmetry (cubic) but I thought that it will still be 'near cubic' symmetry and that I could still treat it as cubic and get meaningful information by looking at those lines of symmetry (gamma to X, X to M, M to Gamma, Gamma to R, R to X and R to M). I expected to see 4x repeats along each line of symmetry due to using the supercell instead of the unit cell, but that's not what I got. Also I'm realizing that since I have an even number of super cells 0.5 0.5 0.5 is not the same point as it would be for a unit cell. Does anyone have a source for how to address this or do I just need to go through all of the geometry shifting in K Space manually? I found a couple of old links but they're all broken.
  • asked a question related to Condensed Matter Physics
Question
12 answers
Superconducting electron pairs occur on the Fermi surface, where the electron kinetic energy is a few eV. The binding energy of paired electrons is usually a few 10-3 eV, so the electrons seemingly cannot remain paired. However, pairs are stable until thermal fluctuations destroy them. Is the situation paradoxical?
Relevant answer
Answer
As a naive observation on R. Monnier's argument of available final states:
The Fermi-distribution is a continuous function of energy (even though values below 2 kT become very small), so by the argument that pairs break when states become available under the Fermi level, the superconducting phase transition should be continuous, not jump-like as typically observed.
  • asked a question related to Condensed Matter Physics
Question
9 answers
I am quite confused. I know that parallel planes do have the same Miller indices. However, as you can see from the attached XRD pattern, there is (003) family of planes having different Miller indices. Why so? What actually happening here
Relevant answer
Answer
you are right, all these peaks arise from parallel planes.
The 003 planes are paralell to the 006 planes, and paralled to the 009 planes etc, but parallel to the 002 and the 001 planes as well.
However their interplanar distances are different und thus the diffraction peaks show up at different angles
Alltogethers all these planes are multiple order planes of the 001 plane.
Please remind the Bragg law:
n*lambda= 2*d*sin(theta)
You may rewrite this equation as:
lambda= 2*d/n * sin(theta)
one also has for any d(h,k,l)/n = d(nh,nk,nl)
You may check the validity of this equation for all crystal systems.
The formulas for dhkl are for example summarized in the attachment, taken from the Klug&Alexander book on 'X-Ray Diffraction Procedures'...
Ggod luck and
best regards
G.M.
  • asked a question related to Condensed Matter Physics
Question
4 answers
The thermal energy, destroying the superconducting gap, may be considered as energy of pair breaking. In other words, that is the energy, which the electron pair absorbs for breaking. The absorbable thermal energy of particle (here the electron pair) depends on the number of independent motions (degrees of freedom) of the particle. The factor 3.5 corresponds to a free particle with cylindrical symmetry, vibrating along its own cylinder axis. Does it mean the factor 3.5 of the thermal pair breaking is a thermodynamic consequence from the real-space-configuration of the electron pair?
Relevant answer
Answer
You are most welcome, Prof. Stanislav Dolgopolov
In our group that works on unconventional superconductors with strontium, we have found the zero energy gap parameter Δ0 to be between 0.1 and 1 meV to reproduce well-established theories in the triplet compound strontium ruthenate using a Wigner distribution approach.
Check please for the one last publication, link to the DOI for the manuscript:
But in HTSC with doped nonmagnetic strontium, Δ0 can be between 10 meV and almost 70 meV if the nonmagnetic disorder is high using the same Wigner distribution approach.
Check our last electronic publication, the link we the manuscript in the DOI
Best Regards.
  • asked a question related to Condensed Matter Physics
Question
3 answers
Usually in a gas atoms and molecules are in random order. So I have doubts that whether gas has a specific crystal structure.
Relevant answer
Answer
Of course not.
  • asked a question related to Condensed Matter Physics
Question
7 answers
"In crystalline solids, where the wave vector k becomes a good quantum number, the wave function can be viewed as a mapping from the k-space to a manifold in the Hilbert space (or in its projection), and hence the topology becomes relevant to electronic states in solids" - This is a statement in the introduction of Yoichi Ando's comprehensive review on topological insulators. Ref: Ando Y., Topological insulator materials, J. Phys. Soc. Japan, (2013), 82, 102001.
I find it difficult to understand why k being a good quantum number allows for the wavefunction to be viewed as a mapping from k-space to a manifold in Hilbert space. I would appreciate insights on the statement given in quotes. Other approaches to explaining why Hilbert space topology becomes relevant to electronic states in TI are also welcome. Thanks in advance.
Relevant answer
Answer
Well, they lead to the localization of light which are transversal waves and have a sort of elliptical polarization, among many other interesting phenomena, Prof. Stam Nicolis
You can check for example:
Best Regards.
  • asked a question related to Condensed Matter Physics
Question
6 answers
It is well known that non-zero negative exchange energy indicates that a singlet state of electrons is energetically more favorite than a triplet one. Sufficiently strong thermal fluctuations destroy any magnetic spin order, so singlet and triplet order becomes equiprobable in the crystal. Hence below a certain temperature (say T*) the energy gain of the singlet order may be larger than the destroying thermal energy, and then preferred singlet pairs become stable. Thus the pairing energy is the difference between two energies:
E1. Energy of the stable singlet;
E2. Energy of the state without spin ordering, where singlet/triplet are equiprobable.
Note: we consider conduction electrons, i.e. electronic wave packets are much larger than lattice constant. So the result is not related with antiferromagnetic order.
This simple logic shows the electron pairing can be derived only from the non-zero negative exchange energy. Feel free to comment or to correct the result.
Relevant answer
Answer
Thank you for the reference. The spin-mediated interaction between electrons takes place. However, for the superconductivity the spin interaction seems to be too weak, because the distance between electrons in a pair may be up to 100 nm, much larger than distances of spin-mediated forces.
  • asked a question related to Condensed Matter Physics
Question
15 answers
1. In a TI surface state/edge state, each k state exists in pairs. The Dirac cone in a 3D-TI has a -k state for every +k state.
2. Due to spin-momentum locking caused by high Spin Orbit Coupling (SOC), the -k state will possess opposite spin to that of +k.
Am I correct in understanding that the combination of these two conditions is what makes the system be termed as a time reversal symmetry protected system? That is, k needs a -k (Kramer degeneracy), and the -k state is opposite in spin also. Hence a TR operation completely reverses the state.
If yes, my question is the following:
What physical properties (band structure, crystal structure) of a system causes a material to possess the Kramer degeneracy? That is, physically what causes a material's band structure to possess k states in pairs?
But, the kramer degeneracy theorem is defined as: 'every eigen state in a TIME REVERSAL SYMMETRIC system with half integer spin will have at least one other degenerate eigen state'. This definition makes it seem like TRS is one of the requirements for the kramer degeneracy.
I am confused about which is the cause and which is the effect here? Does TRS cause the Kramer degeneracy? Or is the presence of the Kramer degeneracy along with spin-momentum locking causing the system to be called time reversal symmetry protected?
Relevant answer
Answer
A good review to complement the answer by Prof. Stam Nicolis on time reversal symmetry is the classical book by Prof. A. Messiah, the chapter on symmetries and invariance, in the old separate version it was chapter XV, Dr. Abhirami Saminathan
Best Regards.
  • asked a question related to Condensed Matter Physics
Question
27 answers
Do you consider yourself a real scientist in your field?
As for me, I don't because I don't know the answer of many basic questions in solid-state physics. For instance, from what's the energy origin of orbitalizing electrons? Is is the thermal energy at T>0 or some sort of quantum energy or both? What's exactly the group velocity of orbitalizing electronic waves and its relation to the ground state energy and thermal energy near T=0. I know there exist so many formal definitions of all the above terms! But is the exact relation between them? In particular, the quasi-free electrons in the conduction band (at T>0) what is exactly the nature of their (so-called) velocity in equilibrium, in the inter-collisional paths (between successive scattering with atoms )? Is is just their thermal velocity? or combination of this thermal velocity with some sort of quantum energy?
Relevant answer
Answer
Agree on that point, Prof. Waldemar Łasica
Best Regards.
  • asked a question related to Condensed Matter Physics
Question
9 answers
I would like to explain my question with the following illustrative situation. In general, when we apply pressure to the crystalline materials, the following situation arise. Pressure systematically alters the bond length, lattice parameter, volume, effective hybridization, electron density, crystal field splitting, and tunes some strong spin-orbit coupling (SOC) strength.
However, I am not able to get any direct mathematical relationship between pressure and SOC.
Is there any direct mathematical relationship between pressure and SOC of the material? If possible, could you please explain me ? If you know any relevant paper or book, could you please suggest it to me?.
Actually, I have been doing a lot of literature related to this. So far, I did not get any relevant papers that discuss the direct relationship between the pressure and SOC of the material.
Your valuable explanation, suggestion, and guidance will be very useful to our research works. Thank you very much in advance.
Relevant answer
Answer
You are most welcome, Prof. Rajaji Vincent
  • asked a question related to Condensed Matter Physics
Question
2 answers
I am trying to calculate Band structure for the electrode in Siesta. It is a supercell as it should be. Can any one tell me how to unfold the degenerate bands in band structure plot so that I can compare it with transmission?
Relevant answer
Answer
  • asked a question related to Condensed Matter Physics
Question
4 answers
Most conventional theories of superconductivity (SC) use the second quantization notation (SQN) where all electrons are assumed indistinguishable, every electron can take every state in the momentum space. However, a sample shows that SQN is insensitive for supercurrent description.
For clarity we consider only 4 electrons (which may belong to arbitrary many-body system): a non-dissipative singlet pair (e1,e2) and two normal (dissipative) electrons e3, e4 . We investigate two cases, A and B:
A. The non-dissipative pair (e1,e2) is permanent. Then an initial non-zero momentum Px of the pair is also permanent. Obviously, this permanent Px is a supercurrent;
B. The non-dissipative pair (e1,e2) is not permanent, i.e. a recombination is possible: e1, e2 become normal, e3, e4 become non-dissipative and back. But at every time moment there are one non-dissipative pair and two normal electrons:
(e1,e2)singlet + e3 + e4 <=> e1 + e2 + (e3,e4)singlet
In case B the initial non-zero momentum of the pair (e1,e2) dissipates, because the electrons e1,e2 become periodically dissipative and there is no external force to give to the newly created pair (e3,e4) exactly the same momentum Px, which the pair (e1,e2) had. So the momentum Px of the system dissipates and the current vanishes. Thus non-permanent pairs cannot keep a supercurrent (otherwise the momentum conservation law is violated; the atom lattice took the momentum Px of the broken pair e1,e2, hence Px of the new pair (e3,e4) must be zero). Notable is the fact that both cases A and B are identical in SQN due to equal occupation numbers (in both cases there are exactly two normal and two SC electrons). However, the case A is superconducting and the case B is dissipative. The cause of the paradox is the indistinguishability of electrons.
Thus the SQN principle of indistinguishability of particles is insensitive to the supercurrent description, we should consider the normal and SC-electrons as distinguishable, i.e. non-exchangeable in the momentum space particles.
So far nobody could plausibly reconcile this paradox and conventional theories of SC.
Relevant answer
Answer
Yes, a qualitatively correct description is a precursor for an accurate approach. For the above considered problem a brief description is : in superconductors there are two electronic components (SC electrons, normal electrons), distinguishable in the momentum space. That is every electron belongs to its component as long as the SC state persists, any interchange between components is impossible. Mathematically this mean we should introduce two Fock spaces or two sets of quantum states, which don’t overlap (i.e. there are not common states).
One important consequence: all derivations of conventional theories should be revised within the two-space-approach.
  • asked a question related to Condensed Matter Physics
Question
4 answers
Dear all,
I want to prepare a gap(less then 10 um) for my experment. I know silicon wafer might be a good choice(it`s easy to cleave). I try to cleave a wafer with diamond tip(or diamond pen) and push them together, but the effect was not ideal. The gap is about 20um, and the gap isn't straight enough. I want to know is there any way to get a um gap? I can also try other materials. I look forward to your suggestions.
Relevant answer
Answer
Lithography technique using X-ray can make even sub-micron size gaps in Si wafers. Till now many electronics industries are using the lithography technique only for making Chips.
  • asked a question related to Condensed Matter Physics
Question
6 answers
Hello everyone, I am currently working on a Heusler alloy system which has a non-collinear magnetic order as reported by a earlier study. I intend to further explore this non-collinear magnetic state. It would be really helpful if someone can suggest me some properties that can be investigated theoretically in order to see if it has a potential use in spintronics devices or if it has some kind of other applications. I am using VASP. Thank you.
Relevant answer
Answer
you should also go for spin hall conductivity and anomalous hall conductivity calculations using Wannier90 package. other important property is exchange parameters, the strength, range and type of exchange parameters plays an important role in determining Curie temperature and macroscopic magnetism of the materials.
  • asked a question related to Condensed Matter Physics
Question
1 answer
hello everyone, I am currently working on a Heusler alloy that has a very low spin polarization (below 10%). Can it still be used in spintronics devices? (usually higher spin polarization is preferred for spintronics application). Also, I should add that the antiferromagnetic state of the compound has almost twice the Magnetocrystalline Anisotropy Energy as compared to the ferromagnetic state (which is THE energetically stable state for the compound).
Relevant answer
Answer
The question is very interesting, thank you, Dear Bhargab Kakati
Probably the citing references to the following research article could have a partial answer, unfortunately, I do not have access to it.
Best Regards.
  • asked a question related to Condensed Matter Physics
Question
4 answers
Hello all, I am currently working on a system that contains Pt, and when I've plotted the 2D ELF pattern, this kind of plot was obtained. So, is there any kind of explanation for these kinds of plots?
Relevant answer
Answer
Thank you all for your valuable insights. I think I should have mentioned above that I have not got any high ELF value in the core of Pt atom for the antiferromagnetic arrangement of the system (done with same pseudopotential). The above-mentioned case was for ferromagnetic arrangement. Here I am attaching the AFM elf 2D pattern.
  • asked a question related to Condensed Matter Physics
Question
3 answers
Hello. Can anyone please tell me how to set INCAR/POSCAR for AF1, AF2 magnetic structure calculation, introducing different magnetic ordering for different planes? I tried making the POSCAR file using VESTA but I am not being able to turn off the symmetry completely. I thought if I turn off the symmetry I can set MAGMOM for each individual atom of a certain plane in the INCAR file but I am not being able to do so while creating the POSCAR file using VESTA. VESTA automatically fills up each corner position of the unit cell due to symmetry and therefore when I set MAGMOM in the INCAR file, one value of MAGMOM covers all the corner points, hence not being able to set different value/direction for different corner atoms
Relevant answer
Answer
you initialize the magnetic moments of the atoms with the MAGMOM tag in VASP. For calculation with collinear spins, you can set ISPIN=2 in order to run spin-polarized calculations with the MAGMOM tag to the initial values of the magnetic moments for each atom. you must set the MAGMOM tag to
MAGMOM = 1.0 -1.0 -1.0 1.0 knowing, you should follow the same order as in the POSCAR file.
If you want to calculate non-collinear magnetic systems (or if you include spin-orbit coupling in the calculations with LSORBIT=.TRUE. , by which the LNONCOLLINEAR tag is automatically set to true also) then you should specify the x y and z components of the magnetic moment for each atom, again in the same order as the atoms appear in the POSCAR file.
  • asked a question related to Condensed Matter Physics
Question
17 answers
A newest Nature paper E. T. Mannila et al, "A superconductor free of quasiparticles for seconds" https://www.nature.com/articles/s41567-021-01433-7 shows that superconducting (SC) pairs persist at least for seconds. The measurement device detects single pair-breaking-events for a large pair population, so the average life time of each pair is much longer than a few seconds (probably, many hours). Thus, every pair hosts its electrons a long time. In most SC-experiments worldwide, the measurement time is much shorter than the life time of the long-hosting SC-states, therefore we can assert that the SC-electrons and normal electrons are non-exchangeable during the measurement, i.e. the SC-electrons do not hop into normal states (at least during the resistance measurement). If so, then the SC-electrons and normal electrons are distinguishable and the superconductor has two distinguishable electronic components: (i) SC-electrons; (ii) normal electrons.
Each of the distinguishable components has its own set of quantum states, its own one-particle-wavefunction, its own Fock space, although the components are overlapped in the real space.
Mainstream theories of superconductivity (BCS etc.) operate within one electronic component and don't take into account this distinguishable 2-component-nature. Should the theories be updated according to the newest finding ?
Relevant answer
Answer
A simple answer, electrons at the Fermi level are given by the equation pF = ℏ kF if they are around the Fermi surface then there is a linear approximation to that equation: δp = ℏ ( k - kF ), i.e., which is consistent for most normal metals and serves well for the Fermi-Dirac distribution, the Sommerfeld expansion, the Fermi liquid theory and the concept of quasiparticles.
In addition, electrons are fermions which means they can only occupy one state with one value for spin +/- 1/2, therefore a Fermi Dirac distribution in momentum space implicitly shows that electrons are separate in momentum space if they are treated using QM and for 3 approximations, the free, the quasi-free, and the tight-binding ones.
Best Regards.